Download as pdf or txt
Download as pdf or txt
You are on page 1of 36

MANAGEMENT ADVISORY SERVICES WORKING CAPITAL FINANCE

WORKING CAPITAL POLICY Cash $ 50 $ 30


. Which of the following statements is incorrect about working capital policy? (M) Marketable securities 0 20
a. A company may hold a relatively large amount of cash if it anticipates uncertain sales Accounts receivable 40 20
levels in the coming year. Inventories 100 50
b. Credit policy has an impact on working capital since it has the potential to influence sales Net fixed assets 500 500
levels and the speed with which cash is collected. Total assets $690 $620
c. The cash budget is useful in determining future financing needs.
d. Holding minimal levels of inventory can reduce inventory carrying costs and cannot lead Spontaneous liabilities $ 30 $ 10
to any adverse effects on profitability.
Short-term debt 50 0
e. Managing working capital levels is important to the financial staff since it influences
Long-term debt 300 300
financing decisions and overall profitability of the firm. Brigham
Common equity 310 310
Total claims $690 $620
Conservative Working Capital Policy
*. Compared to other firms in the industry, a company that maintains a conservative working From this data we may conclude that (M)
capital policy will tend to have a (D) a. Ski Lifts has a working capital financing policy of exactly matching asset and liability
a. Greater percentage of short-term financing. maturities.
b. Greater risk of needing to sell current assets to repay debt. b. Ski Lifts’ working capital financing policy is relatively aggressive; that is, the company
c. Higher ratio of current assets to fixed assets. finances some of its permanent assets with short-term discretionary debt.
d. Higher total asset turnover. RPCPA 0595 c. Ski Lifts follows a relatively conservative approach to working capital financing; that is,
some of its short-term needs are met by permanent capital.
1. As a company becomes more conservative with respect to working capital policy, it would tend d. Without income statement data, we cannot determine the aggressiveness or conservatism
to have a(n) of the company’s working capital financing policy. Brigham
a. Increase in the ratio of current liabilities to noncurrent liabilities.
b. Decrease in the operating cycle. Aggressive Working Capital Policy
c. Decrease in the quick ratio. *. A firm following an aggressive working capital strategy would (M)
d. Increase in the ratio of current assets to noncurrent assets. CMA 1290 1-23 a. Hold substantial amount of fixed assets.
b. Minimize the amount of short-term borrowing.
2. As a company becomes more conservative in its working capital policy, it would tend to have c. Finance fluctuating assets with long-term financing.
a(n) d. Minimize the amount of funds held in very liquid assets. RPCPA 1091
A. Decrease in its acid-test ratio.
B. Increase in the ratio of current liabilities to noncurrent liabilities. 3. The working capital financing policy that subjects the firm to the greatest risk of being unable to
C. Increase in the ratio of current assets to units of output. meet the firm’s maturing obligations is the policy that finances (E)
D. Increase in funds invested in common stock and a decrease in funds invested in a. Fluctuating current assets with long-term debt.
marketable securities. CMA 1296 1-8 b. Permanent current assets with long-term debt.
c. Permanent current assets with short-term debt.
. Ski Lifts Inc. is a highly seasonal business. The following summary balance sheet d. Fluctuating current assets with short-term debt. CMA 1295 1-2
provides data for peak and off-peak seasons (in thousands of dollars):
Peak Off-peak

RPCPA, AICPA, CMA & CIA EXAMINATION QUESTIONS Page 1 of 36


MANAGEMENT ADVISORY SERVICES WORKING CAPITAL FINANCE

4. Since Marsh, Inc. is experiencing a sharp increase in sales activity and a steady increase in
production, the management of Marsh has adopted an aggressive working capital policy.
Therefore, the company's current level of net working capital
A. Would most likely be the same as in any other type of business condition as business
cycles tend to balance out over time.
B. Would most likely be lower than under other business conditions in order that the
company can maximize profits while minimizing working capital investment.
C. Would most likely be higher than under other business conditions so that there will be
sufficient funds to replenish assets.
D. Would most likely be higher than under other business conditions as the company's profits
are increasing. CMA 0689 1-11

Aggressive vs. Conservative Working Capital Policy


5. Clay Corporation follows an aggressive financing policy in its working capital management
while Lott Corporation follows a conservative financing policy. Which one of the following
statements is correct?
A. Clay has a low ratio of short-term debt to total debt while Lott has a high ratio of short-
term debt to total debt.
B. Clay has a low current ratio while Lott has a high current ratio.
C. Clay has less liquidity risk while Lott has more liquidity risk.
D. Clay's interest charges are lower than Lott's interest charges. CMA 1284 1-22

RPCPA, AICPA, CMA & CIA EXAMINATION QUESTIONS Page 2 of 36


MANAGEMENT ADVISORY SERVICES WORKING CAPITAL FINANCE

WORKING CAPITAL MANAGEMENT Effect of Transactions on Working Capital


Appropriate Level of Working Capital 9. Starrs Company has current assets of $300,000 and current liabilities of $200,000. Starrs
6. Determining the appropriate level of working capital for a firm requires (E) could increase its working capital by the (E)
a. Evaluating the risks associated with various levels of fixed assets and the types of debt A. Prepayment of $50,000 of next year's rent.
used to finance these assets. B. Refinancing of $50,000 of short-term debt with long-term debt.
b. Changing the capital structure and dividend policy for the firm. C. Purchase of $50,000 of temporary investments for cash.
c. Maintaining short-term debt at the lowest possible level because it is ordinarily more D. Collection of $50,000 of accounts receivable. CMA 1293 1-19
expensive than long term debt.
d. Offsetting the profitability of current assets and current liabilities against the probability of 10. Starrs has current assets of 300,000 and current liabilities of 200,000. Starrs could increase its
technical insolvency. working capital by the
e. Maintaining a high proportion of liquid assets to total assets in order to maximize the A. Prepayment of 50,000 of next year's rent.
return on total investments. CMA 0689 1-10, 0696 1-16, RPCPA 0596 B. Refinancing of 50,000 of short-term debt with long-term debt.
C. Acquisition of land valued at 50,000 through the issuance of common shares.
7. Which of the following statements about current asset management is most correct? (E) D. Purchase of 50,000 of financial assets held for trading for cash. CMA 1293 1-19
a. A positive net float means that a company has more cash available for its use than the
amount shown in the company’s books. *. Which of the following transactions causes an increase in working capital?
b. Use of a lockbox reduces the possibility that petty cash will be lost. a. Sale of merchandise on credit at a price above cost.
c. Depreciation has an impact on the cash budget. b. Sale of marketable securities at a price below cost.
d. Statements a and c are correct. Brigham c. Collection of an account receivable.
d. Return to supplier of defective merchandise purchased on credit. Full credit allowed by
Working Capital supplier. RPCPA 1075
Working Capital Defined
*. The amount of long-term capital that is made to revolve in conducting operations and serves . Other things held constant, which of the following will cause an increase in working capital?
as the lifeblood of the company (E) (M)
a. Paid-up capital c. Working capital a. Cash is used to buy marketable securities.
b. Net worth d. None of these RPCPA 0588 b. A cash dividend is declared and paid.
c. Merchandise is sold at a profit, but the sale is on credit.
8. The working capital is the difference between d. Long-term bonds are retired with the proceeds of a preferred stock issue.
A. Current assets and current liabilities. C. Total assets and total liabilities. e. Missing inventory is written off against retained earnings. Brigham
B. Fixed assets and fixed liabilities. D. Equity and cash. CMA 0692 1-25
11. Which one of the following would increase the working capital of a firm?
3. Net Working Capital is the: A. Cash payment of payroll taxes payable.
A. Difference between short-term assets and short term liabilities B. Purchase of a new plant financed by a 20-year mortgage.
B. Difference between long-term assets and long term liabilities C. Cash collection of accounts receivable.
C. Difference between long-term assets and short term liabilities D. Refinancing a short-term note payable with a two-year note payable. CMA 1294 1-15
D. None of the above B&M
12. If a firm increases its cash balance by issuing additional shares of common stock, working
capital (E)
a. Remains unchanged and the current ratio remains unchanged.
RPCPA, AICPA, CMA & CIA EXAMINATION QUESTIONS Page 3 of 36
MANAGEMENT ADVISORY SERVICES WORKING CAPITAL FINANCE

b. Increases and the current ratio remains unchanged. a. Cash flow control c. Maximizing sales
c. Increases and the current ratio decreases. b. Cash surplus investment d. Obtaining financing services RPCPA 0590
d. Increases and the current ratio increases. CMA 1294 1-30
2. Firms would need to hold zero cash when:
13. The following transactions occurred during a company's first year of operations: A. Transaction-related needs are greater than cash inflows
I. Purchased a delivery van for cash B. Transaction-related needs are less than cash inflows
I. Borrowed money by issuance of short-term debt C. Transaction-related needs are not perfectly synchronized with cash inflows
III. Purchased treasury stock D. Transaction-related needs are perfectly synchronized with cash inflows B&M
Which of the items above caused a change in the amount of working capital?
16. Which of the following statements is most correct? (E)
A. I only. C. II and III only.
B. I and II only. D. I and III only. CIA 0593 IV-28 a. A good cash management system would minimize disbursement float and maximize
collections float.
Comprehensive b. If a firm begins to use a well-designed lockbox system, this will reduce its customers’ net
14. All of the following statements in regard to working capital are correct except (M) float.
a. Current liabilities are an important source of financing for many small firms. c. In the early 1980s, the prime interest rate hit a high of 21 percent. In 2000 the prime rate
b. Profitability varies inversely wit liquidity. was considerably lower. That sharp interest rate decline has increased firms’ concerns
c. The hedging approach to financing involves matching maturities of debt with specific about the efficiency of their cash management programs.
d. If a firm can get its customers to permit it to pay by wire transfers rather than having to
financing needs.
write checks, this will increase its net float and thus reduce its required cash balances.
d. Financing permanent inventory buildup with long-term debt is an example of an
e. A firm that has such an efficient cash management system that it has positive net float
aggressive working capital policy. CMA 0696 1-29
can have a negative checkbook balance at most times and still not have its checks
bounce. Brigham
CASH MANAGEMENT
Function
Motives for Holding Cash
15. When managing cash and short-term investments, a corporate treasurer is primarily
17. According to John Maynard Keynes, the three major motives for holding cash are for
concerned with (E)
A. Transactional, psychological, and social purposes.
a. Maximizing rate of return.
B. Speculative, fiduciary, and transactional purposes.
b. Minimizing taxes.
C. Speculative, social, and precautionary purposes.
c. Investing in Treasury bonds since they have no default risk.
D. Transactional, precautionary, and speculative purposes. CMA 1286 1-32
d. Investing in common stock due to the dividend exclusion for federal income tax purposes.
e. Liquidity and safety. CMA 1295 I-12
*. A precautionary motive for holding excess cash is (E)
a. To enable a company to meet the cash demands from the normal flow of business
1. Determining the appropriate target cash balance involves assessing the trade-off between:
activity.
A. Income and diversification
b. To enable a company to avail itself of a special inventory purchase before prices rise to
B. The benefit and cost of liquidity
higher levels.
C. Balance sheet strength and transaction needs
c. To enable a company to have cash to meet emergencies that may arise periodically.
D. All of the above B&M
d. To avoid having to use the various types of lending arrangements available to cover
projected cash deficits. RPCPA 0595
*. Which of the following is not a major function in cash management? (E)

RPCPA, AICPA, CMA & CIA EXAMINATION QUESTIONS Page 4 of 36


MANAGEMENT ADVISORY SERVICES WORKING CAPITAL FINANCE

18. The amount of cash that a firm keeps on hand in order to take advantage of any bargain
purchases that may arise is referred to as its 13. Most large firms hold a cash balance greater than most models imply because:
A. Transactions balance. C. Precautionary balance. A. It is too difficult to estimate the costs of security transactions
B. Compensating balance. D. Speculative balance. CIA 1194 IV-15 B. Banks are compensated by account balances for payment of services
C. Corporations have few bank accounts and it is difficult to manage their cash
25. Which of the following statements concerning zero balance accounts is not correct? D. Cash is costless and need not be managed closely B&M
A. They are set up to handle disbursement activity
B. The account has a minimum amount at all times 14. We should expect cash balances to increase when:
C. Checks are automatically transferred into the account as checks presented for payment A. The transaction costs of buying or selling interest-bearing securities increase
D. The transfer is automatic and involves an accounting entry only B. Interest rates increase
E. The master and the zero balance account locate at the same bank B&M C. Sales volume falls
D. Uncertainty about day-to-day or week-to-week cash flows decreases B&M
26. Which of the following is used to control disbursements?
A. Concentration banking C. Lock-box system Methods of Accelerating Cash Collections
B. Zero-balance account D. Fedwire B&M 32. Banks generally offer the following cash management services:
A. Processing checks C. Running lock-boxes
19. All of the following are valid reasons for a business to hold cash and marketable securities B. Transferring funds D. All of the above B&M
except to
A. Satisfy compensating balance requirements. 33. Banks generally offer the following services:
B. Maintain adequate cash needed for transactions. A. Processing checks C. Providing advice and references
C. Meet future needs. B. Running lock-boxes D. All of the above B&M
D. Earn maximum returns on investment assets. CMA 0694 1-22
Disbursement Float
. Which of the following statements is most correct? (M) 15. Checks written by the firm are said to generate:
a. The cash balances of most firms consist of transactions, compensating, precautionary, A. Availability float C. Disbursement float
and speculative balances. The total desired cash balance can be determined by B. Ledger float D. Book float B&M
calculating the amount needed for each purpose and then summing them together.
b. The easier a firm’s access to borrowed funds the higher its precautionary balances will be, 16. The difference between bank cash and book cash is called:
in order to protect against sudden increases in interest rates. A. Disbursement float C. Availability float
c. For some firms, holding highly liquid marketable securities is a substitute for holding cash B. Net float D. None of the above B&M
because the marketable securities accomplish the same objective as cash. Brigham
d. Firms today are more likely to rely on cash than on reserve borrowing power or
Maximizing Net Float
marketable securities for speculative purposes because of the need to move quickly. 20. Which of the following statements is most correct? (E)

a. A cash management system that minimizes collections float and maximizes disbursement
12. A large firm may hold substantial cash balances because: float is better than one with higher collections float and lower disbursement float.
A. These balances are required by the bank b. A cash management system that maximizes collections float and minimizes disbursement
B. The company may have accounts in many different banks float is better than one with lower collections float and higher disbursement float.
C. The company may have a very decentralized organization c. The use of a lockbox is designed to minimize cash theft losses. If the cost of the lockbox
D. All of the above B&M is less than theft losses saved, then the lockbox should be installed. Brigham
RPCPA, AICPA, CMA & CIA EXAMINATION QUESTIONS Page 5 of 36
MANAGEMENT ADVISORY SERVICES WORKING CAPITAL FINANCE

d. Other things held constant, a firm will need a smaller line of credit if it can arrange to pay Draft
its bills by the 5th of each month than if its bills come due uniformly during the month. 22. A working capital technique that delays the outflow of cash is (E)
A. Factoring. C. A lock-box system.
. Which of the following statements is most correct? (M) B. A draft. D. Electronic funds transfer. CMA 1293 1-21
a. Poor synchronization of cash flows that results in high cash management costs can be
partially offset by increasing disbursement float and decreasing collections float. 23. A working capital technique that increases the payable float and therefore delays the outflow
b. The size of a firm’s net float is primarily a function of its natural cash flow synchronization of cash is (E)
and how it clears its checks. A. Concentration banking. C. Electronic Data Interchange (EDI).
c. Lockbox systems are used mainly for security purposes as well as to decrease the firm’s
B. A draft. D. A lockbox system. CMA 1296 1-5
net float.
d. If a firm can speed up its collections and slow down its disbursements, it will be able to
reduce its net float. Automated Clearing House (ACH) Electronic Transfer
e. A firm practicing good cash management and making use of positive net float will bring its 27. Electronic cash transfers offer several advantages, including:
check book balance as close to zero as possible, but must never generate a negative A. A low marginal transactions cost C. Easy automation of record-keeping
book balance. Brigham B. A reduced float D. All of the above B&M

Lock-Box System 31. A check processed through ACH (Automated Clearing House):
21. The most common cash management technique used to speed up collections is: A. Will clear immediately C. Will take two or three days to clear
A. Concentration banking C. Lock-boxes B. Will clear the same day D. None of the above B&M
B. Wire transfers D. In-house processing B&M
24. An automated clearing house (ACH) electronic transfer is a(n)
23. By getting closer to the source of payment, lock-boxes can be used to reduce: a. Electronic payment to a company’s account at a concentration bank.
A. Availability or clearing float C. In-house processing float b. Check that must be immediately cleared by the Federal Reserve Bank.
B. Mail float D. Disbursement float B&M c. Computer-generated deposit ticket verifying deposit of funds. CMA 0694 1-23)
d. Check-like instrument drawn against the payor and not against the bank.
21. A lock-box system
A. Reduces the need for compensating balances.
B. Provides security for late night deposits.
C. Reduces the risk of having checks lost in the mail.
D. Accelerates the inflow of funds. CMA 1293 1-20

. A lockbox plan is most beneficial to firms that (M)


a. Send payables over a wide geographic area.
b. Have widely disbursed manufacturing facilities.
c. Have a large marketable securities account to protect.
d. Hold inventories at many different sites.
e. Make collections over a wide geographic area. Brigham

RPCPA, AICPA, CMA & CIA EXAMINATION QUESTIONS Page 6 of 36


MANAGEMENT ADVISORY SERVICES WORKING CAPITAL FINANCE

*. The following practices will impact the cash flow of the company: (E)
a. Sales personnel are unequivocally responsible for collecting their credit sales. 26. A typical firm doing business nationally cannot expect to accelerate its cash inflow by
b. Sales commissions are based on collected invoices. A. Establishing multiple collection centers throughout the country.
c. Statement of accounts receivable are reconciled with customers and regularly sent for B. Employing a lockbox arrangement.
confirmation. RPCPA 0594 C. Initiating controls to accelerate the deposit and collection of large checks. CMA 1283 1-23
d. Automatic transfer of funds is arranged with banks regarding deposits of branches. D. Maintaining compensating balances rather than paying cash for bank services.

Wire Transfer Baumol’s Model of Cash Balances


28. The fastest but most expensive way to transfer surplus funds from the local deposit bank to 3. Baumol's model of cash balances states that: Q = [(2 x T x C./i]^0.5 where T = annual cash
the concentration bank is: disbursement, C = cost per sale of T-bills, i = interest rate. What is Q?
A. A lock-box system D. An in-house processing float system A. The number of times per annum bill should be sold
B. A mail float system E. An availability float system B. The average holding of bills
C. A wire transfer B&M C. The amount of T-bills that should be sold at any one time
D. The minimum holding of cash B&M
Concentration Banking
22. Which of the following is a way for companies to speed up collections? 9. The Baumol model determines the optimal cash balance by:
A. Remote disbursing C. Baumol model A. Balancing total costs against opportunity costs
B. Concentration banking D. All of the above B&M B. Minimizing total costs of holding cash against trading securities costs
C. Balancing trading securities costs against total costs
Comprehensive D. Minimizing total costs less trading costs B&M
*. Which of the following actions would not be consistent with good management? (M)
a. Increased synchronization of cash flows. 10. Concerning the Baumol model, which of the following is not correct (all other things equal)?
b. Minimize the use of float. A. The optimum cash balance is higher at higher interest rates
c. Maintaining an average cash balance equal to that required as a compensating balance B. The optimum cash balance is higher at higher fixed order costs
or that which minimizes total cost. C. The optimum cash balance is higher at higher total cash requirement
d. Use of checks and drafts in disbursing funds. RPCPA 0595 D. All of the above are correct B&M

*. A company’s management is concerned about the large bank overdraft, which it wishes to 11. The Baumol cash balance model is limited by:
reduce over the budget period of one year. Which one of the four items below will not A. Assuming the cash flows are variable across the period
necessarily result in a lower bank overdraft? (E) B. A smooth disbursement rate and now cash inflows over the period
a. Reducing bad debts. C. Having a safety stock set to zero
b. Taking longer credit from suppliers without any loss of discounts. D. Both B and C B&M
c. Reducing wastage and loss through damage of regularly-used stock items.
d. Reducing unit costs of production. RPCPA 0592 Cash Conversion Cycle
27. An enterprise plans to produce a new product, which will typically be sold to other firms on
25. Methods of accelerating cash collections include all of the following except (E) credit. The cash conversion cycle resulting from this new product can be measured as the
A. Decentralized collections. C. Compensating balances. length of time from
B. Electronic funds transfers. D. Lockbox systems. CMA 0689 1-12 A. Cash purchases of raw materials to the collection of accounts receivable.

RPCPA, AICPA, CMA & CIA EXAMINATION QUESTIONS Page 7 of 36


MANAGEMENT ADVISORY SERVICES WORKING CAPITAL FINANCE

B. Cash purchases of raw materials to the time the final product is completed. C. Risk-profitability trade-off considerations.
C. Cash purchases of raw materials to the sale of the product. D. Flotation cost considerations. CMA 0684 1-5
D. When the product is completed to the sale of the product. CIA 1193 IV-52
32. Some managers express the opinion that “cash management problems are nothing more than
28. Helena Furnishings wants to sharply reduce its cash conversion cycle. Which of the following inventory problems.” They then proceed to use cash management module, such as the EOQ
steps would reduce its cash conversion cycle? (E) model to determine the
a. The company increases its average inventory without increasing its sales. a. Credit and collection policies.
b. The company reduces its DSO. b. Marketable securities level.
c. The company starts paying its bills sooner, which reduces its average accounts payable c. Proper relationship between current assets and current liabilities.
without reducing its sales. d. Proper blend of marketable securities and cash. CMA 1286 1-33
d. Statements a and b are correct. Brigham
33. Determining the amount and timing of conversions of marketable securities to cash is a critical
. Ignoring cost and other effects on the firm, which of the following measures would tend to element of a financial manager's performance. In terms of the rate of return forgone on
reduce the cash conversion cycle? (M)
converted securities and the cost of such transactions, the optimal amount of cash to be raised
a. Maintain the level of receivables as sales decrease.
b. Buy more raw materials to take advantage of price breaks. by selling securities is
c. Take discounts when offered. A. Inversely related to the rate of return forgone and directly related to the cost of the
d. Forgo discounts that are currently being taken. transaction.
e. Offer a longer deferral period to customers. Brigham B. Directly related to the rate of return forgone and directly related to the cost of the
transaction.
29. Which of the following actions are likely to reduce the length of a company’s cash conversion C. Directly related to the rate of return forgone and inversely related to the cost of the
cycle? (M) transaction.
a. Adopting a just-in-time inventory system that reduces the inventory conversion period. D. Inversely related to the rate of return forgone and inversely related to the cost of the
b. Reducing the average days sales outstanding (DSO) on its accounts receivable. transaction. CIA 0593 IV-52
c. Reducing the amount of time the company takes to pay its suppliers.
d. Statements a and b are correct. Brigham 34. The economic order quantity (EOQ) formula can be adapted in order for a firm to determine
the optimal mix between cash and marketable securities. The EOQ model assumes all of the
Optimal Mix Between Cash and Short-term Investments following except
30. When managing cash and short-term investments, a corporate treasurer is primarily a. The cost of a transaction is independent of the dollar amount of the transaction and
concerned with (E) interest rates are constant over the short run.
A. Maximizing rate of return. b. An opportunity cost is associated with holding cash, beginning with the first dollar.
B. Minimizing taxes. c. The total demand for cash is known with certainty.
C. Investing in Treasury bonds since they have no default risk. d. Cash flow requirements are random. CMA 0689 1-15
D. Liquidity and safety. CMA 1295 1-12
Comprehensive
31. When a company is evaluating whether the ratio of cash and marketable securities to total *. The following are desirable in cash management except: (E)
assets should be high or low, its decision will be based upon a. Cash is collected at the earliest time possible.
A. Financial leverage considerations. b. Most sales are on cash basis and receivables are aged “current”
B. Operating leverage considerations. c. Post-dated checks are not deposited on time upon maturity.

RPCPA, AICPA, CMA & CIA EXAMINATION QUESTIONS Page 8 of 36


MANAGEMENT ADVISORY SERVICES WORKING CAPITAL FINANCE

d. All sales are properly receipted and promptly deposited intact. RPCPA 0593 35. Which of the following are criteria for selection among securities available for a marketable
securities portfolio? (M)
*. In cash management, which of the following statements is false? (M) A. Default risk, taxability, and relative yields.
a. Capital costs, delinquency costs, and default costs are costs associated with cash B. Planning, major investment decisions, and interaction with capital markets.
management. C. The operating, payment, and cash conversion cycles. CIA 0589 IV-52
b. Short costs, long costs, and procurement costs are costs associated with optimal cash D. Inventory conversion, receivables conversion, and payable deferral periods.
balance model approach
c. Obtaining financing services and controlling cash flow are some of the major functions of . Which of the following is not a situation that might lead a firm to hold marketable securities?
cash management. (M)
d. Funds sourcing and custodianship must be done at the lowest possible cost, where a. The firm has purchased a fixed asset that will require a large write-off of depreciable
excess funds must be invested for a return that is best in the market. RPCPA 1090 expense.
b. The firm must meet a known financial commitment, such as financing an ongoing
MARKETABLE SECURITIES construction project.
Criteria c. The firm must finance seasonal operations.
*. The criteria that should be considered in investing surplus cash (E) d. The firm has just sold long-term securities and has not yet invested the proceeds in
a. Safety of the company’s funds c. Yield on the principal earning assets. Brigham
b. Liquidity of the principal d. All of the above RPCPA 0588
Money Market Instruments
7. The following are money market instruments except:
A. T-bills C. Commercial paper
B. Federal agency discount notes D. Preferred stocks B&M

8. The three money market securities with the greatest volume of business are:
A. Treasury bills, commercial paper, repurchase agreements
B. Negotiable CDs, federal agency discount notes, T-bills
C. Commercial paper, bankers' acceptances, tax-exempt municipal notes
D. Federal agency discount notes, repurchase agreements, medium-term notes B&M

Marketable Securities Portfolio


. Which of the following statement completions is most correct? If the yield curve is upward
sloping, then a firm’s marketable securities portfolio, assumed to be held for liquidity purposes,
should be (M)
a. Weighted toward long-term securities because they pay higher rates.
b. Weighted toward short-term securities because they pay higher rates.
c. Weighted toward U. S. Treasury securities to avoid interest rate risk.
d. Weighted toward short-term securities to avoid interest rate risk.
e. Balanced between long- and short-term securities to minimize the effects of either an
upward or a downward trend in interest rates. Brigham

RPCPA, AICPA, CMA & CIA EXAMINATION QUESTIONS Page 9 of 36


MANAGEMENT ADVISORY SERVICES WORKING CAPITAL FINANCE

Treasury Bills
36. Which security is most often held as a substitute for cash? (E) 10. Negotiable CDs are issued by:
A. Treasury bills. C. Gold. A. US Government C. Banks
B. Common stock. D. Aaa corporate bonds. CMA 1289 1-14 B. Federal agencies D. Corporations B&M

9. For which of the following investments is there a very active secondary market? Commercial Paper
A. Medium-term notes C. US Treasury bills 41. The best example of a marketable security with minimal risk would be (D)
B. Commercial paper D. Repurchase agreements B&M A. Municipal bonds.
B. The common stock of a Aaa rated company.
37. The marketable securities with the least amount of default risk are C. The commercial paper of a Aaa rated company.
A. Federal government agency securities. C. Repurchase agreements. D. Stock options of a Aaa rated company. CMA 0688 1-15
B. U.S. Treasury securities. D. Commercial paper. CMA 0691 1-11
EuroCommercial Paper
38 Of the following four investments, is considered to be the safest. (E) 14. Commercial papers sold in the international markets are called:
a. commercial paper d. Treasury bonds A. Negotiable commercial papers C. MTNs
b. corporate bonds e. U. S. Agency issues B. EuroCommercial papers D. None of the above B&M
c. Treasury bills
Floating Rate Preferred Stock
39. Which of the following financial instruments can be traded in international money markets? 16. Floating-rate preferred stock offers competitive rates of return with traditional money-market
A. Mortgages. C. U.S. Treasury bills. instruments but:
B. Preferred shares. D. U.S. Treasury bonds. CIA 1195 IV-65 A. Is not rated by Moody's or Standard & Poor's
B. Still provides the corporate investor with the tax exclusion on dividend income
Repurchase Agreement C. Has a fixed rate of dividend income
15. A repurchase agreement occurs when: D. Offers a highly competitive trading market B&M
A. A company agrees to buy back its commercial paper before maturity
B. A bank depositor agrees, in advance, to re-invest money in a negotiable certificate of 17. A tax-paying corporation would prefer to invest short-term money in:
deposit A. Preferred stock C. Common stock
C. An investor buys part of a government security dealer's inventory and simultaneously B. Floating-rate preferred stock D. Long-term bonds B&M
agrees to sell it back
D. The federal government agrees to buy T-bills B&M 21. Even though the dividend rate on a floating-rate preferred stock is floating to keep in line with
interest rates, the instrument still suffers from risk such as:
Negotiable Certificates of Deposit A. A thin market causing potential principal risk and liquidity concerns
40. Which one of the following is not a characteristic of a negotiable certificate of deposit? B. The risk of downgrades from the narrow range of issuers
Negotiable certificates of deposit (D) C. The impact of tax law changes, which may reduce the after-tax value of the instrument
A. Have a secondary market for investors. D. All of the above B&M
B. Are regulated by the Federal Reserve System.
C. Are usually sold in denominations of a minimum of $100,000. CMA 0691 1-12
D. Have yields considerably greater than bankers' acceptances and commercial paper.

RPCPA, AICPA, CMA & CIA EXAMINATION QUESTIONS Page 10 of 36


MANAGEMENT ADVISORY SERVICES WORKING CAPITAL FINANCE

Money Market Mutual Funds D. The Eurodollar market is outside the direct control of the U.S. monetary authorities and
42. In smaller businesses in which the management of cash is but one of numerous functions has lower costs. CIA 0595 IV-59
performed by the treasurer, various cost incentives and diversification arguments suggest that
surplus cash should be invested in 3. There is a difference between "Eurodollar" or "international dollar" rate and short-term interest
A. Commercial paper. C. Money market mutual funds. rate in the US because:
B. Bankers' acceptances. D. Corporate bonds. CMA 1280 1-1 A. Banks are not subject to reserve requirements on international dollars
B. Banks need not insure international dollar deposits with the FDIC
Eurodollar Deposit C. Both A and B
2. "Eurodollars" or "international dollars" are: D. None of the above B&M
A. Dollar deposits in banks outside the US
B. Dollars deposited in the US by foreigners Variable Rate Demand Bond
C. Dollars held by foreign governments 11. A variable rate demand bond (VRDB):
D. None of the above B&M A. Is a long-term security
B. Has interest payments linked to the level of short-term interest rates
44. US dollars deposited in a German bank are called: C. May periodically be sold back to the issuer at face value
A. Deutsche dollars C. Eurodollars D. Is tax-exempt
B. American depositing receipts D. None of the above B&M E. All of the above B&M

43. Of the following, a characteristic of Eurobonds is that they are Not a Proper Investment for Idle Cash
A. Always denominated in Eurodollars. 46. All of the following are alternative marketable securities suitable for investment except
B. Always sold in some country other than the one in whose currency the bond is A. U.S. Treasury bills. C. Commercial paper.
denominated. B. Eurodollars. D. Convertible bonds. CMA 0694 1-25
C. Sold outside the country of the borrower but are denominated in the currency of the
country in which the issue is sold. *. Which of the following investments is not likely to be a proper investment for temporary idle
D. Generally issued as registered bonds. CIA 1195 IV-66 cash? (E)
a. Initial public offering of an established profitable conglomerate.
44. Which of the following statements does not properly describe a Eurodollar deposit? b. Commercial paper.
A. Eurodollar deposits are U.S. dollar deposits in banks outside of the U.S. c. Treasury bills.
B. Eurodollar deposits are outside the direct control of the U.S. monetary authorities. d. Treasury bonds due within one year. RPCPA 0595
C. Eurodollar deposit rates tend to be lower than domestic U.S. rates on equivalent
instruments. CIA 0594 IV-58 Short Selling
D. Interest rates on Eurodollar deposits are tied to the London Interbank Offer Rate (LIBOR). 47. The term “short selling” is the
a. Selling of a security that was purchased by borrowing money from a broker.
45. Interest rates received by depositors on Eurodollar deposits tend to be higher than domestic b. Selling of a security that is not owned by the seller.
U.S. rates on equivalent instruments because c. Selling of all the shares you own in a company in anticipation that the price will decline
A. Borrowers pay higher rates than domestic U.S. rates on equivalent instruments. dramatically. CMA 1294 1-16
B. The deposits involve different currencies. d. Betting that a stock will increase by a certain amount within a given period of time.
C. Eurodollar deposits are for smaller amounts.

RPCPA, AICPA, CMA & CIA EXAMINATION QUESTIONS Page 11 of 36


MANAGEMENT ADVISORY SERVICES WORKING CAPITAL FINANCE

Riding the Yield Curve 49. The average collection period for a firm measures the number of days (E)
*. In the process of investing of surplus cash, the term “riding the yield curve” refers to (E) a. After a typical credit sale is made until the firm receives the payment.
a. Diversifying securities portfolio so that the firm has an equal balance of long-term versus b. It takes a typical check to “clear” through the banking system.
short-term securities. c. Beyond the end of the credit period before a typical customer payment is received.
b. Swapping different maturities of similar quality debt securities in order to obtain higher d. Before a typical account becomes delinquent. CMA 1295 1-4
yield.
c. Purchasing only the longest maturities for given rates of return. . Which of the following statements is most correct? (M)
d. Adherence to the liquidity preference theory of securities investment. RPCPA 0596 a. If a firm’s volume of credit sales declines then its DSO will also decline.
b. If a firm changes its credit terms from 1/20, net 40 days, to 2/10, net 60 days, the impact
RECEIVABLES MANAGEMENT on sales can’t be determined because the increase in the discount is offset by the longer
. Which of the following statements is most correct? (M) net terms, which tends to reduce sales.
a. A firm that makes 90 percent of its sales on credit and 10 percent for cash is growing at a c. The DSO of a firm with seasonal sales can vary. While the sales per day figure is usually
rate of 10 percent annually. If the firm maintains stable growth it will also be able to based on the total annual sales, the accounts receivable balance will be high or low
maintain its accounts receivable at its current level, since the 10 percent cash sales can depending on the season.
be used to manage the 10 percent growth rate. d. An aging schedule is used to determine what portion of customers pay cash and what
b. In managing a firm’s accounts receivable it is possible to increase credit sales per day yet portion buy on credit.
still keep accounts receivable fairly steady if the firm can shorten the length of its e. Aging schedules can be constructed from the summary data provided in the firm’s
collection period. financial statements. Brigham
c. If a firm has a large percentage of accounts over 30 days old, it is a sign that the firm’s
receivables management needs to be reviewed and improved. Aging of Accounts Receivable
d. Since receivables and payables both result from sales transactions, a firm with a high 50. An aging of accounts receivable measures the
receivables-to-sales ratio should also have a high payables-to-sales ratio. Brigham a. Ability of the firm to meet short-term obligations.
b. Average length of time that receivables have been outstanding. CMA 1290 1-22
Monitoring Receivables c. Percentage of sales that have been collected after a given period of time.
. Analyzing days sales outstanding (DSO) and the aging schedule are two common methods for d. Amount of receivables that have been outstanding for given lengths of time.
monitoring receivables. However, they can provide erroneous signals to credit managers when
(E) . Which of the following statements is most correct? (M)
a. Customers’ payments patterns are changing. a. Other things held constant, the higher a firm’s days sales outstanding (DSO), the better its
b. Sales fluctuate seasonally. credit department.
c. Some customers take the discount and others do not. b. If a firm that sells on terms of net 30 changes its policy and begins offering all customers
d. Sales are relatively constant, either seasonally or cyclically. Brigham terms of 2/10, net 30, and if no change in sales volume occurs, then the firm’s DSO will
probably increase.
Days Sales Outstanding c. If a firm sells on terms of 2/10, net 30, and its DSO is 30 days, then its aging schedule
48. An enterprise's receivables collection period is equal to would probably show some past due accounts.
A. The inventory conversion period. d. Statements a and c are correct. Brigham
B. The cash conversion cycle.
C. The day's sales outstanding. Credit & Collection Policy
D. The inventory divided by average daily sales. CIA 0597 IV-27 *. The goal of credit policy is to (E)
a. Extend credit to the point where marginal profits equal marginal costs.
RPCPA, AICPA, CMA & CIA EXAMINATION QUESTIONS Page 12 of 36
MANAGEMENT ADVISORY SERVICES WORKING CAPITAL FINANCE

b. Minimize bad debt losses. e. Seasonal dating with terms 2/15, net 30 days, with April 1 dating, means that if the original
c. Minimize collection expenses. sale took place on February 1st, the customer can take the discount up until March 15th,
d. Maximize sales. RPCPA 0597 but must pay the net invoice amount by April 1st. Brigham

*. It is held that the level of accounts receivable that the firm has or holds reflects both the Average Gross Receivables Balance
volume of a firm’s sales on account and a firm’s credit policies. Which one of the following 53. Which of the following represents a firm’s average gross receivables balances?
items is not considered as part of the firm’s credit policies? (M) I. Days’ sales in receivables x accounts receivable turnover.
a. The minimum risk group to which credit should be extended. II. Average daily sales x average collection period.
b. The extent (in terms of money) to which a firm will go to collect an account. III. Net sales ÷average gross receivables. (E)
c. The length of time for which credit is extended. a. I only. c. II only.
d. The size of the discount that will be offered. RPCPA 1095 b. I and II only. d. II and III only. CMA 1296 1-18

51. The one item listed below that would warrant the least amount of consideration in credit and Receivable Turnover
collection policy decisions is the 54. A high turnover of accounts receivable, which implies a very short days-sales outstanding,
A. Quality of accounts accepted. C. Cash discount given. CMA 0684 1-3 could indicate that the firm
B. Quantity discount given. D. Level of collection expenditures. A. Has a relaxed (lenient) credit policy.
B. Offers small discounts.
52. When a company analyzes credit applicants and increases the quality of the accounts C. Uses a lockbox system, synchronizes cash flows, and has short credit terms.
rejected, the company is attempting to (M) D. Has an inefficient credit and collection department. Gleim
A. Maximize sales. C. Increase the average collection period.
B. Increase bad-debt losses. D. Maximize profits. CMA 1286 1-34 55. A decrease in the firm's receivable turnover ratio means that (D)
A. It is collecting credit sales more quickly than before.
. If easing a firm’s credit policy lengthens the collection period and results in a worsening of the B. It is collecting credit sales more slowly than before.
aging schedule, then why do firms take such actions? (E) C. Sales have gone up.
a. It normally stimulates sales. D. Inventories have gone up. Gleim
b. To meet competitive pressures.
c. To increase the firm’s deferral period for payables. 56. Accounts receivable turnover will normally decrease as a result of (E)
d. Statements a and b are correct. Brigham a. The write-off of an uncollectible account (assume the use of the allowance for doubtful
accounts method).
. Which of the following statements is most correct? (M) b. A significant sales volume decrease near the end of the accounting period.
a. If credit sales as a percentage of a firm’s total sales increases, and the volume of credit c. An increase in cash sales in proportion to credit sales.
sales also increases, then the firm’s accounts receivable will automatically increase. d. A change in credit policy to lengthen the period for cash discounts. CMA 0690 4-14
b. It is possible for a firm to overstate profits by offering very lenient credit terms that
encourage additional sales to financially “weak” firms. A major disadvantage of such a Credit Terms
policy is that it is likely to increase uncollectible accounts.
57. An organization would usually offer credit terms of 2/10, net 30 when
c. A firm with excess production capacity and relatively low variable costs would not be
a. The organization can borrow funds at a rate exceeding the annual interest cost.
inclined to extend more liberal credit terms to its customers than a firm with similar costs
that is operating close to capacity. b. The orgnization can borrow funds at a rate less than the annual interest cost.
d. Firms use seasonal dating primarily to decrease their DSO. c. The cost of capital approaches the prime rate.

RPCPA, AICPA, CMA & CIA EXAMINATION QUESTIONS Page 13 of 36


MANAGEMENT ADVISORY SERVICES WORKING CAPITAL FINANCE

d. Most competitors are offering the same terms and the organization has a shortage of *. The level of accounts receivable will most likely increase as (M)
cash. CMA 0691 1-7 a. Cash sales increase and number of says sales.
b. Credit limits are expanded, credit sales increase, and credit terms remain the same.
58. Which one of the following statements is most correct if a seller extends credit to a purchaser c. Credit limits are expanded, cash sales increase, and aging of the receivables is
for a period of time longer than the purchaser’s operating cycle? The seller (E) improving.
a. Will have a lower level of accounts receivable than those companies whose credit period d. Cash sales increase, current receivables ratio to past due increases, credit limits remain
is shorter than the purchaser’s operating cycle. the same. RPCPA 0594
b. Is, in effect, financing more than just the purchaser’s inventory needs.
c. Can be certain that the purchaser will be able to convert the inventory into cash before 60. A change in credit policy has caused an increase in sales, an increase in discounts taken, a
payment is due. decrease in the amount of bad debts, and a decrease in the investment in accounts
d. Has no need for a stated discount rate or credit period. CMA 1296 1-13 receivable. Based upon this information, the company’s (E)
A. Average collection period has decreased.
Change in Credit Policy B. Percentage discount offered has decreased.
59. An increase in sales resulting from an increased cash discount for prompt payment would be C. Accounts receivable turnover has decreased.
expected to cause (M) D. Working capital has increased. CMA 1296 1-6
A. An increase in the operating cycle.
B. An increase in the average collection period. 61. A change in credit policy has caused an increase in sales, an increase in discounts taken, a
C. A decrease in the cash conversion cycle. reduction of the investment in accounts receivable, and a reduction in the number of doubtful
D. A decrease in purchase discounts taken. CMA 1285 1-6 accounts. Based on this information, we know that: (E)
a. Net profit has increased.
*. If a firm had been extending trade credit on a 2/10, net/30 basis, what change would be b. The average collection period has decreased.
expected on the balance sheet of its customer if the firm went to a net cash 30 policy? (M) c. Gross profit has declined.
a. Increased payables and increased bank loan. d. The size of the discount offered has decreased.
b. Increased receivables. e. The bad debt loss percentage has increased. RPCPA 1095, CMA 1289 1-15
c. Decreased receivables.
d. Decrease in cash. RPCPA 0596 *. The credit and collection policy of Amargo Co. provides for the imposition of credit block when
the credit line is exceeded and/or the account is past due. During the month, because of the
campaign to achieve volume targets, the general manager has waived the credit block policy
in a number of instances involving big volume accounts. The likely effect of this move is (M)
a. Deterioration of aging of receivables only.
b. Increase in the level of receivables only.
c. Deterioration of aging and increase in the level of receivables.
d. Decrease in collections during the month the move was done. RPCPA 1094

62. The sales manager at Ryan Company feels confident that, if the credit policy at Ryan’s were
changed, sales would increase and, consequently, the company would utilize excess capacity.
The two credit proposals being considered are as follows:
Proposal A Proposal B
RPCPA, AICPA, CMA & CIA EXAMINATION QUESTIONS Page 14 of 36
MANAGEMENT ADVISORY SERVICES WORKING CAPITAL FINANCE

Increase in sales $500,000 $600,000 D. All of the above B&M


Contribution margin 20% 20%
Bad debt percentage 5% 5% Factoring
Increase in operating profits $75,000 $90,000 64. Factoring is the (E)
Desired return on sales 15% 15% A. Selling of accounts receivable by one company to another.
Currently, payments terms are net 30. The proposed payment terms for Proposal A and B. Selling of inventory by one company to another.
Proposal B are net 45 and net 90, respectively. An analysis to compare these two proposals C. Conversion of accounts receivable to bad debt on financial statements for accounts that
for the change in credit policy would include all of the following factors except the are long overdue. CIA 0590 IV-50
a. Cost of funds for Ryan. D. Adjustment of inventories on financial statements for supplies that have become obsolete.
b. Current bad debt experience.
c. Impact on the current customer base of extending terms to only certain customers. Pledging
d. Bank loan covenants on days’ sales outstanding. CMA 0697 1-14 65. Short-term borrowing using accounts receivable and characterized by the fact that the lender
not only has a claim against the receivables but also has recourse against the borrower is
*. A strict credit and collection policy is in place in Star Co. As Finance Director you are asked to A. Factoring. C. Selling.
advise on the propriety of relaxing the credit standards in view of stiff competition in the B. Pledging. D. Hedging. CIA 0589 IV-53
market. Your advise will be favorable if (E)
a. The competitor will do the same thing to prevent lost sales. SHORT-TERM FINANCING
b. there is a decrease in the distribution level of your product, and a more aggressive stance Short-term & Long-term Financing
in necessary to retain market share. *. Debt capital be it long-term or short-term in nature, can be raised from a number of different
c. The projected margin from increased sales will exceed the cost of carrying the sources including
incremental receivables. a. Trade credit c. All of the above
d. The account receivable level is improving, so the company can afford the carrying cost of b. Bank loan d. None of the above RPCPA 1091
receivables. RPCPA 0594
66. In general, as a company increases the amount of short-term financing relative to long-term
63. A company serves as a distributor of products by ordering finished products once a quarter financing, the
and using that inventory to accommodate the demand over the quarter. If it plans to ease its A. Greater the risk that it will be unable to meet principal and interest payments.
credit policy for customers, the amount of products ordered for its inventory every quarter will B. Leverage of the firm increases.
be C. Likelihood of having idle liquid assets increases.
A. Increased to accommodate higher sales levels. D. Current ratio increases. CMA 0688 1-17
B. Reduced to offset the increased cost of carrying accounts receivable.
C. Unaffected if safety stock is part of the current quarterly order. . Firms generally choose to finance temporary assets with short-term debt because (M)
D. Unaffected if the JIT inventory control system is used. CIA 0593 IV-53 a. Matching the maturities of assets and liabilities reduces risk.
b. Short-term interest rates have traditionally been more stable than long-term interest rates.
Assignment c. A firm that borrows heavily long-term is more apt to be unable to repay the debt than a
35. In a loan arranged through the assignment of accounts receivable the lender: firm that borrows heavily short-term.
A. Accepts the actual receivable to be collected d. The yield curve has traditionally been downward sloping. Brigham
B. Has a lien on the receivables and recourse to the borrower e. Sales remain constant over the year, and financing requirements also remain constant.
C. Assumes full risk of default
RPCPA, AICPA, CMA & CIA EXAMINATION QUESTIONS Page 15 of 36
MANAGEMENT ADVISORY SERVICES WORKING CAPITAL FINANCE
67. Which of the following statements is most correct? (M) 70. Which one of the following financial instruments generally provides the largest source of short-
a. Under normal conditions, a firm’s expected ROE would probably be higher if it financed term credit for small firms?
with short-term rather than with long-term debt, but the use of short-term debt would A. Installment loans. C. Trade credit.
probably increase the firm’s risk. B. Commercial paper. D. Mortgage bonds. CMA 1295 1-9
b. Conservative firms generally use no short-term debt and thus have zero current liabilities.
c. A short-term loan can usually be obtained more quickly than a long-term loan, but the cost 71. Which one of the following statements about trade credit is correct?
of short-term debt is likely to be higher than that of long-term debt. a. Not an important source of financing for small firms.
d. If a firm that can borrow from its bank buys on terms of 2/10, net 30, and if it must pay by b. A source of long-term financing to the seller.
Day 30 or else be cut off, then we would expect to see zero accounts payable on its c. Subject to risk of buyer default.
balance sheet. d. Usually an inexpensive source of external financing. CMA 1296 1-12
e. If one of your firm’s customers is “stretching” its accounts payable, this may be a nuisance
but does not represent a real financial cost to your firm as long as the firm periodically 72. The correct equation for calculating the approximate percentage cost, on an annual basis, of
pays off its entire balance. Brigham not taking trade discounts is CIA 1195 IV-53
A. Discount % 360
. Which of the following statements is most correct? (M) x
100 - Discount % Days credit is outstandin g - Discount period
a. Under normal conditions the shape of the yield curve implies that the interest cost of
short-term debt is greater than that of long-term debt, although short-term debt has other B. Discount % 360
x
advantages that make it desirable as a financing source. 100 Days credit is outstandin g - Discount period
b. Flexibility is an advantage of short-term credit but this is somewhat offset by the higher C. 100  Discount % 360
flotation costs associated with the need to repeatedly renew short-term credit. x
c. A short-term loan can usually be obtained more quickly than a long-term loan but the Discount % Days credit is outstandin g - Discount period
penalty for early repayment of a short-term loan is significantly higher than for a long-term D. Discount % Days credit is outstandin g - Discount period
loan.
x
100 - Discount % 360
d. Statements about the flexibility, cost, and riskiness of short-term versus long-term credit
are dependent on the type of credit that is actually used. Brigham 73. A company obtaining short-term financing with trade credit will pay a higher percentage
e. Short-term debt is often less costly than long-term debt and the major reason for this is financing cost, everything else being equal, when (D)
that short-term debt exposes the borrowing firm to much less risk than long-term debt. A. The discount percentage is lower.
B. The items purchased have a higher price.
Trade Credit C. The items purchased have a lower price.
68. Which one of the following provides a spontaneous source of financing. (E) D. The supplier offers a longer discount period. CIA 0597 IV-51
a. Accounts payable. c. Accounts receivable.
b. Mortgage bonds. d. Debentures. CMA 1295 1-15 74. Which one of the following statements concerning cash discount is correct? (E)
A. The cost of not taking a 2/10, net 30 cash discount is usually less than the prime rate.
69. Which one of the following is a spontaneous source of financing? B. With trade terms of 2/15, net 60, if the discount is not taken, the buyer receives 45 days of
A. Notes payable. C. Prepaid interest. free credit. CMA 1295 1-7
B. Long-term debt. D. Trade credit. CMA 1289 1-20 C. The cost of not taking the discount is higher for terms of 2/10, net 60 than for 2/10, net 30.
D. The cost of not taking a cash discount is generally higher than the cost of a bank loan.

RPCPA, AICPA, CMA & CIA EXAMINATION QUESTIONS Page 16 of 36


MANAGEMENT ADVISORY SERVICES WORKING CAPITAL FINANCE

75. A small retail business would most likely finance its merchandise inventory with
A. Commercial paper. C. A line of credit. 23. Which of the following describes short-term bank loans?
B. A terminal warehouse receipt loan. D. A chattel mortgage. CMA 0689 1-17 A. If unsecured, banks often require borrower to "clean up" the loan for 1 month in the year
B. Often secured by commercial paper
76. Which of the following statements is most correct? (M) C. Almost never secured by accounts receivable
a. Accruals are an expensive way to finance working capital. D. Cannot be from an international bank because Federal Reserve System regulations
b. A conservative financing policy is one in which the firm finances all of its fixed assets with prohibit Eurodollar borrowing
long-term capital and part of its permanent current assets with short-term, E. None of the above B&M
nonspontaneous credit.
c. If a company receives trade credit under the terms 2/10 net 30, this implies the company Term Loan
has 10 days of free trade credit. 24. Which of the following statements describes bank term loans?
d. Statements a and b are correct. Brigham A. Typical maturity is between 2 and 4 years
B. Usually repaid in level amounts over the term of the loan
77. Merkle, Inc. has a temporary need for funds. Management is trying to decide between not C. Interest rate is usually fixed
taking discounts from one of their three biggest suppliers, or a 14.75% per annum renewable D. Borrower is often obliged to have the bank mange its corporate cash accounts B&M
discount loan from its bank for 3 months. The suppliers' terms are as follows:
Fort Co. 1/10, net 30 25. Which of the following statements describe bank term loans?
Riley Manufacturing Co. 2/15, net 60 A. Typical maturity is between 1 and 2 years
Shad, Inc. 3/15, net 90 B. Usually repaid in increasing amounts over the term of the loan
Using a 360-day year, the cheapest source of short-term financing in this situation is (D) C. Interest rate is usually fixed
A. The bank. C. Riley Manufacturing Co. D. Borrower is often obliged to maintain compensating balance B&M
B. Fort Co. D. Shad, Inc. CMA 1283 1-25
Interest rates
Bank Loans 78. Short-term interest rates are
. Which one of the following aspects of banks is considered most relevant to businesses when a. Usually lower than long-term rates.
choosing a bank? (M) b. Usually higher than long-term rates.
a. Convenience of location. c. Lower than long-term rates during periods of high inflation only.
b. Competitive cost of services provided. d. Not significantly related to long-term rates. CMA 0691 1-5
c. Size of the bank’s deposits.
d. Experience of personnel. 79. The prime rate is the (D)
e. Loyalty and willingness to assume lending risks. Brigham A. Size of the commitment fee on a commercial bank loan.
B. Effective cost of a commercial bank loan.
22. Which of the following describes short-term bank loans? C. Effective cost of commercial paper.
A. Often secured by a factoring agent D. Rate charged on business loans to borrowers with high credit ratings. CMA 0688 1-18
B. Almost never secured by inventory
C. Often prearranged as a line of credit 80. The prime lending rate of commercial banks is an announced rate and is often understated
D. Cannot be from an international bank because of Federal Reserve System regulations from the viewpoint of even the most credit-worthy firms. Which one of the following
E. None of the above B&M requirements always results in a higher effective interest rate?

RPCPA, AICPA, CMA & CIA EXAMINATION QUESTIONS Page 17 of 36


MANAGEMENT ADVISORY SERVICES WORKING CAPITAL FINANCE

A. A floating rate for the loan period. Effective Rate


B. A covenant that restricts the issuance of any new unsecured bonds during the existence 84. Discounted interest is based on the borrowed amount but is paid in advance. The formula for
of the loan. calculating the discounted interest rate for a 1-year loan is (E) Gleim
C. The imposition of a compensating balance with an absolute minimum that cannot be met Interest Interest
by current transaction balances. A. C.
Borrowed amount Borrowed amount - Interest
D. The absence of a charge for any unused portion in the line of credit. CMA 1280 1-3
Interest
B. D. None of the answers are correct.
Nominal Interest Rate Average borrowed amount
81. A one year, $20,000 loan with a 10% nominal interest rate provides the borrower with the use
of <List A> if interest is charged on a <List B> basis. (E) Collateral
CIA 0595 IV-50 A. B. C. D. *. In assessing the loan value of inventory, a banker will normally be concerned about the portion
List A $18,000 $20,000 $20,000 $22,000 of inventory that is work-in-process because (E)
List B Simple Simple Discount Discount a. WIP inventory is relatively easy to sell because it does not represent a raw material or a
finished product.
Compensating Balance b. WIP inventory usually has the highest loan value of the different inventory types.
82. A minimum checking account balance that a firm must maintain with a commercial bank is a c. WIP generally has the lowest marketability of the various types of inventories.
A. Transactions balance. C. Precautionary balance. d. WIP represents a lower investment by a corporation as opposed to other types of
B. Compensating balance. D. Speculative balance. CIA 1190 IV-49 inventories. RPCPA 0596

83. A compensating balance 34. Firms provide the following as security for short-term loans:
A. Compensates a financial institution for services rendered by providing it with deposits of A. Accounts receivables C. Securities
funds. CMA 0688 1-13 B. Inventories D. All of the above B&M
B. Is used to compensate for possible losses on a marketable securities portfolio.
C. Is a level of inventory held to compensate for variations in usage rate and lead time. Syndicated Loan
D. Is an amount paid by financial institutions to compensate large depositors. 30. When banks have to make large loans, they form a group of banks for the purpose of making
the loan. The group is called a:
. Which of the following statements is most correct? (M) A. Bank holding company C. Golden umbrella
a. Compensating balance requirements apply only to businesses, not to individuals. B. Syndicate D. Conglomerate B&M
b. Compensating balances are essentially costless to most firms, because those firms would
normally have such funds on hand to meet transactions needs anyway. Loan Sales by Commercial Banks
c. If the required compensating balance is larger than the transactions balance the firm 32. Loan sales by commercial banks may take the form of:
would ordinarily hold, then the effective cost of any loan requiring such a balance is A. Loan assignments D. All of the above
increased. B. Loan participations E. A and B
d. Banks are prohibited from earning interest on the funds they force businesses to keep as C. Loan syndications B&M
compensating balances. Brigham
33. Loan participations are different from syndicated loans in that:
A. The lead bank provides a "certificate of participation" to each bank
B. Each participating bank has a separate loan agreement with the borrower

RPCPA, AICPA, CMA & CIA EXAMINATION QUESTIONS Page 18 of 36


MANAGEMENT ADVISORY SERVICES WORKING CAPITAL FINANCE

C. The loans are transferred to new lenders


D. None of the above B&M 89. The principal advantage of using commercial paper as a short-term financing instrument is that
it (E)
Banker’s Acceptance A. Is usually cheaper than a commercial bank loan.
85. The credit instrument known as a banker's acceptance B. Is readily available to almost all companies.
A. Calls for immediate payment upon delivery of the shipping documents to the bank's C. Offers security, i.e., collateral, to the lender.
customer and acceptance of goods by the bank. D. Can be purchased without commission costs. CMA 1289 1-22
B. Involves an invoice being signed by the banker upon receipt of goods, after which both
the banker and the seller record the transaction on their respective books. 90. Which one of the following responses is not an advantage to a corporation that uses the
C. Is a time draft payable on a specified date and guaranteed by the bank. commercial paper market for short-term financing?
D. Is a method of sales financing in which the bank retains title to the goods until the buyer a. This market provides more funds at lower rates than other methods provides.
has completed payment. CIA 0596 IV-41 b. The borrower avoids the expense of maintaining a compensating balance with a
commercial bank.
Unsecured Credit c. There are no restrictions as to the type of corporation that can enter into this market.
86. The following forms of short-term borrowing are available to a firm: d. This market provides a broad distribution for borrowing. CMA 0696 1-14
 Floating lien
 Factoring 37. Large companies often raise short-term debt by selling:
 Revolving credit A. Bonds C. Medium term notes
 Chattel mortgages B. Debentures D. Commercial paper B&M
 Bankers' acceptances
 Lines of credit . Which of the following statements concerning commercial paper is incorrect? (E)
 Commercial paper a. Commercial paper is generally written for terms less than 270 days.
The forms of short-term borrowing that are unsecured credit are b. Commercial paper generally carries an interest rate below the prime rate.
A. Floating lien, revolving credit, chattel mortgage, and commercial paper. c. Commercial paper is sold to money market mutual funds, as well as to other financial
B. Factoring, chattel mortgage, bankers' acceptances, and line of credit. institutions and nonfinancial corporations.
C. Floating lien, chattel mortgage, bankers' acceptances, and line of credit. CMA 1286 1-35 d. Commercial paper can be issued by virtually any firm so long as it is willing to pay the
D. Revolving credit, bankers' acceptances, line of credit, and commercial paper. going interest rate. Brigham
e. Commercial paper is a type of unsecured promissory note issued by large, strong firms.
Commercial Paper
87. Short-term, unsecured promissory notes issued by large firms are known as 91. Which of the following statements is incorrect? (E)
A. Agency securities. C. Commercial paper. CMA 0689 1-13 a. Commercial paper can be issued by virtually any firm so long as it is willing to pay the
B. Bankers' acceptances. D. Repurchase agreements. going interest rate.
b. Accruals represent a source of “free” financing in the sense that no explicit interest is paid
88. Commercial paper on these funds.
A. Has a maturity date greater than 1 year. c. A conservative approach to working capital will result in all permanent assets being
B. Is usually sold only through investment banking dealers. financed using long-term securities.
C. Ordinarily does not have an active secondary market.
D. Has an interest rate lower than Treasury bills. CMA 0691 1-10

RPCPA, AICPA, CMA & CIA EXAMINATION QUESTIONS Page 19 of 36


MANAGEMENT ADVISORY SERVICES WORKING CAPITAL FINANCE

d. The risk to the firm of borrowing with short-term credit is usually greater than with long- B. The trade credit. Its effective rate is 20%.
term debt. Added risk can stem from greater variability of interest costs on short-term C. The note. Its effective rate is 17%.
debt. D. The note. Its effective rate is 20%. CIA 1188 IV-54
e. Trade credit is often the largest source of short-term credit. Brigham
95. A short-term bank loan will have a higher effective financing cost if it has which combination of
Commercial Paper Rating characteristics?
12. Which of the following is a commercial paper rating by Moody's? A. A 10% compensating balance and regular interest.
A. Aaa C. P-1 B. A 10% compensating balance and discount interest.
B. Baa D. All of the above B&M C. A 20% compensating balance and regular interest.
D. A 20% compensating balance and discount interest. CIA 1196 IV-52
13. Which of the following is a commercial paper rating by Moody's?
A. P-2 C. Aaa 96. A manufacturing firm wants to obtain a short-term loan and has approached several lending
B. Aa D. None of the above B&M institutions. All of the potential lenders are offering the same nominal interest rate, but the
terms of the loans vary. Which of the following combinations of loan terms will be most
Agency Securities attractive for the borrowing firm?
92. Short-term securities issued by the Federal Housing Administration are known as A. Simple interest, no compensating balance.
A. Agency securities. C. Commercial paper. B. Discount interest, no compensating balance.
B. Bankers' acceptances. D. Repurchase agreements. CMA 0689 1-14 C. Simple interest, 20% compensating balance required.
D. Discount interest, 20% compensating balance required. CIA 0594 IV-51
Secured Short-term Financing
93. An example of secured short-term financing is 97. A company is arranging debt financing for the purchase of a new piece of equipment that has
A. Commercial paper. C. A revolving credit agreement. a 5-year expected useful life. Which of the following alternative financing arrangements has
B. A warehouse receipt. D. Line of credit. CIA 1191 IV-56 the lowest effective annual percentage rate if each has a quoted nominal rate of 9.5%?
A. A 5-year term loan with interest compounded annually.
Inventory Loans B. A 10-year term loan with interest compounded semiannually.
36. The three basic forms of inventory loans include: C. A 5-year term loan with interest compounded quarterly.
A. Blanket inventory lien, field warehouse financing, and line of credit D. A 10-year term loan with interest compounded monthly. CIA 1196 IV-45
B. Blanket inventory lien, line of credit, and trust receipt
C. Blanket inventory lien, field warehouse financing, and trust receipt 98. A company will receive cash from sales in 1 year that can be used to pay for materials. The
D. Field warehouse financing, line of credit, and trust receipt B&M supplier will allow payment in 1 year. If the company pays the supplier immediately, it will
receive a 20% discount off the $100,000 purchase price, but it must borrow the full amount. A
Alternative Financing Arrangements bank has offered the company three alternatives:
94. A corporation is currently experiencing cash-flow problems and has determined that it is in 1. A 1-year loan at 18% with no other fees,
need of short-term credit. It can either use its trade credit on $100,000 of accounts payable 2. A 1-year loan at 15% with the provision that it maintains 20% of whatever amount it
with terms of 1/10, net 30 or a 30-day note with a 20% annual simple interest rate. Which is borrows as noninterest-bearing compensating balances over the life of the loan, or
the best alternative, and what is its effective rate of interest (rounded to a whole percentage 3. A guaranteed line of credit of $100,000 at 17% with the provision that the bank will collect
and using a 360-day year)? (M) a 1% fee on the average amount of unused funds. The company expects to borrow no
A. The trade credit. Its effective rate is 10%. other funds.The company would achieve the lowest cost of financing by (M)

RPCPA, AICPA, CMA & CIA EXAMINATION QUESTIONS Page 20 of 36


MANAGEMENT ADVISORY SERVICES WORKING CAPITAL FINANCE

A. Allowing the supplier to finance the materials and making payment at the end of 1 year.
B. Accepting the 1-year loan at 18% with no other provisions.
C. Accepting the 1-year loan at 15% with the compensating balance provisions.
D. Accepting the guaranteed line of credit at 17% with the fee required on the average
amount of unused funds. CIA 1186 IV-43

RPCPA, AICPA, CMA & CIA EXAMINATION QUESTIONS Page 21 of 36


MANAGEMENT ADVISORY SERVICES WORKING CAPITAL FINANCE

ANSWER EXPALANATIONS current assets with short-term debt is not as risky as financing permanent current assets with
short-term debt.
1. REQUIRED: The true statement about a more conservative working capital policy.
DISCUSSION: (D) A conservative working capital policy results in an increase in working 4. Answer (B) is correct. When a firm has an aggressive working capital policy, management
capital (current assets – current liabilities). It is typified by a reduction in liquidity risk, keeps the investment in working capital at a minimum. Thus, a growing company would want
increasing the current ratio, whether by decreasing current liabilities or increasing current to invest its funds in capital goods and not in idle assets. This policy maximizes return on
assets, minimizes the risk that the company will not be able to meet its obligations as they fall investment at the price of the risk of minimal liquidity.
due. Thus, an increasing ratio of current to noncurrent higher returns on long-term assets in Answer (A) is incorrect because the growing firm is more apt to emphasize production rather
order to guard against short-term cash flow problems. than protecting against technical insolvency by maintaining a high level of working capital.
Answer (A) is incorrect because an increase in current liabilities relative to noncurrent liabilities Answer (C) is incorrect because the company will prefer to expend funds on capital goods.
would increase liquidity risk. Answer (B) is incorrect because a decrease in the normal Answer (D) is incorrect because the company needs its profits to invest in new production
operating cycle permits a lower level of working capital. If assets can be converted to cash equipment in order to grow.
more quickly, current assets can be reduced. Answer (C) is incorrect because a decrease in
the quick ratio signifies that quick assets ( cash, receivables, and marketable securities) are 5. Answer (B) is correct. A conservative working capital management financing policy uses
decreasing relative to current liabilities. permanent capital to finance permanent asset requirements and also some or all of the firm's
seasonal demands. Thus, Lott's current ratio (current assets/current liabilities) will be high
2. Answer (C) is correct. A conservative working capital policy minimizes liquidity risk by since its current liabilities will be relatively low. An aggressive policy entails financing some
increasing working capital (current assets - current liabilities). The result is that the company fixed assets and all the current assets with short-term capital. This policy results in a lower
forgoes the potentially higher returns available from using the additional working capital to current ratio.
acquire long-term assets. A conservative working capital policy is characterized by a higher Answer (A) is incorrect because Clay's aggressive policy would result in more short-term debt,
current ratio (current assets/current liabilities) and acid-test ratio (quick assets/current with attendant renewal problems and high risk. Lott's conservative policy would produce more
liabilities). Thus, the company will increase current assets or decrease current liabilities. A long-term debt or equity financing. Answer (C) is incorrect because Clay is subject to greater
conservative policy finances assets using long-term or permanent funds rather than short-term liquidity risk than Lott since it has greater short-term debt. Hence, it is at greater risk of being
sources. unable to meet its maturing obligations. Answer (D) is incorrect because a more conservative
Answer (A) is incorrect because a decrease in the acid-test ratio suggests an aggressive company would tend to finance by means of equity rather than debt capital. Thus, the more
policy. A conservative company wants a higher acid-test ratio, that is, more liquid assets conservative company would have less interest expense.
relative to liabilities. Answer (B) is incorrect because a conservative company wants working
capital to be financed from long-term sources. Answer (D) is incorrect because a conservative 6. REQUIRED: The true statement about the determination of the appropriate level of working
company seeks more liquid (marketable) investments. capital.
DISCUSSION: (C) A company must maintain a level of working capital sufficient to pay bills
3. REQUIRED: The working capital financing policy that subects a firm to the greatest risk of as they come due. Failure to do so is a technical insolvency and can result in involuntary
being unable to meet maturing obligations. bankruptcy. Unfortunately, holding current assets for purposes of paying bills is not profitable
DISCUSSION: (C) Fluctuating curent assets can often be financed with short-term debt for a company because they usually offer a low return compared with longer-term investments.
because the periodic liquidition of the assets provides funds to pay off the debt. However, Thus, the skillful management of working capital requires a balancing of a firm’s desire for
financing permanent current assets with short-term debt is a risky strategy because the assets profit with its need for adequate liquidity.
may not be liquidated in time to pay off the debt at maturity. Answer (A) is incorrect because management of fixed assets is not a factor in working capital
Answers (A), and (B) are incorrect because it is not particularly risky to finance working captial management. Answer (B) is incorrect because capital structure and dividend policy are
needs from long-term debt sources. Answer (D) is incorrect because financing fluctuating factors involved in capital structure finance, not in working capital financial management.
RPCPA, AICPA, CMA & CIA EXAMINATION QUESTIONS Page 22 of 36
MANAGEMENT ADVISORY SERVICES WORKING CAPITAL FINANCE

Answer (C) is incorrect because short-term debt is usually less expensive than long-term debt. 11. Answer (D) is correct. Working capital equals current assets minus current liabilities.
Answer (E) is incorrect because maintaining a high level of liquid assets is usually not Refinancing a short-term note with a two-year note payable decreases current liabilities, thus
profitable. increasing working capital.
Answer (A) is incorrect because a cash payment of payroll taxes decreases current assets and
7. Statements a and c are correct; therefore, statement d is the appropriate choice. A lockbox current liabilities by equal amounts. Answer (B) is incorrect because buying a new plant with a
speeds collections of receivables; it doesn’t ensure that petty cash will be safe. Although 20-year mortgage has no effect on current assets or current liabilities. Answer (C) is incorrect
depreciation is a noncash expense, it does affect taxes, which are a cash expense. because cash collection of an account receivable increases one current asset and decrease
another by the same amount.
8. REQUIRED: The definition of net working capital.
DISCUSSION: (A) Net working capital is defined as the difference between current assets 12. REQUIRED: The effect on working capital and the current ratio of issuing common stock for
and current liabilities. Working capital is a measure of short-term solvency. cash.
Answer (B) is incorrect because working capital refers to the difference between current DISCUSSION: (D) Working capital equals current assets minus current liabilities. The current
assets and current liabilities; fixed assets are not a component. Answer (C) is incorrect ratio equals current assets divided by current liabilities. Selling stock for cash increases
because total assets and total liabilities are not components of working capital; only current current assets and stockholders’ equity, with no effect on current liabilities. The result is an
items are included. Answer (D) is incorrect because shareholders’ equity is not a component increase in working capital and the current ratio.
of working capital; only current items are included in the concept of working capital. Answers (A), (B) and (C) are incorrect because both working capital and the current ratio
increase.
9. Answer (B) is correct. Working capital is the excess of current assets over current liabilities.
Refinancing a short-term debt with a long-term debt decreases current liabilities, and the result 13. Answer (D) is correct. Working capital is computed by deducting total current liabilities from
is an increase in working capital. total current assets. The purchase of a delivery van for cash reduces current assets and has
Answer (A) is incorrect because a prepayment of expenses does not change current assets or no effect on current liabilities. The borrowing of cash by incurring short-term debt increases
current liabilities. Cash decreases by the same amount that prepaid rent increases. Answer current assets by the same amount as it increases current liabilities; hence, it will have no
(C) is incorrect because the purchase of temporary investments does not affect total current effect on working capital. The purchase of treasury stock decreases current assets but has no
assets; cash is replaced by temporary investments, another current asset. Answer (D) is effect on current liabilities. Thus, the purchases of the van and treasury stock affect working
incorrect because the collection of a receivable has no effect on total current assets. The capital.
receivable is replaced by an equal amount of cash. Answer (A) is incorrect because the purchases of the van and treasury stock affect working
capital. Answer (B) is incorrect because the purchases of the van and treasury stock but not
10. Answer (B) is correct. Working capital is the excess of current assets over current liabilities. the issuance of short-term debt affect working capital. Answer (C) is incorrect because the
Refinancing a short-term debt with a long-term debt decreases current liabilities, and the result purchases of the van and treasury stock but not the issuance of short-term debt affect working
is an increase in working capital. capital.
Answer (A) is incorrect because a prepayment of expenses does not change current assets or
current liabilities. Cash decreases by the same amount that prepaid rent increases. Answer 14. REQUIRED: The false statement about working capital.
(C) is incorrect because the acquisition of land for common shares does not affect either DISCUSSION: (D) Financing permanent inventory build-up which is essentially a long-term
current assets or current liabilities. Answer (D) is incorrect because the purchase of financial investment, with long-term debt is a moderate or conservative working capital policy. An
assets held for trading does not affect total current assets; cash is replaced by the financial aggressive policy uses short-term relatively low-cost debt to finance the inventory buildup. It
assets held for trading, another current asset. focuses on high profitability potential, despite high risk and low liquidity. An aggressive policy
reduces the current ratio and accepts a higher risk of short-term lack of liquidity. Financing

RPCPA, AICPA, CMA & CIA EXAMINATION QUESTIONS Page 23 of 36


MANAGEMENT ADVISORY SERVICES WORKING CAPITAL FINANCE

inventory with long-term debt increases the current ratio and accepts higher borrowing costs in incorrect because the three major motives for holding cash, according to Keynesian
exchange for greater liquidity and lower risk. economics, are for transactional, precautionary, and speculative purposes.
Answer (A) is incorrect because current liabilities, e.g., trade credit is a major source of funds
for small firms. Answer (B) is incorrect because liquid investments tend to have lower returns. 18. Answer (D) is correct. Companies hold cash to facilitate routine transactions, to compensate
Answer (C) is incorrect because matching of asset and liability maturities is a moderate policy banks for providing loans and services, to guard against unforeseen fluctuations in cash flows,
that minimizes risk. The expectation is that cash flows from the assets will be available to and to take advantage of opportunities for bargain purchases. However, firms are more likely
meet obligations for the liabilities. to hold trading securities or rely on easy access to credit to take advantage of bargains or to
guard against unforeseen fluctuations in cash flows.
15. REQUIRED: The primary concern when managing cash and short-term investments. Answer (A) is incorrect because a transactions balance is associated with routine payments
DISCUSSION: (E) Cash and short-term investments are crucial for a firm’s continuing and collections. Answer (B) is incorrect because a compensating balance is the minimum
success. Sufficient liquidity must be available to meet payments as they come due. At the amount on deposit at a bank to compensate the bank for providing loans and other services.
same time, liquid assets are subject to significant control risk. Therefore, liquidity and safety Answer (C) is incorrect because a precautionary balance consists of reserves for unforeseen
are the primary concerns of the treasurer when dealing with a highly liquid assets. Cash and fluctuations in cash flows.
short-term investments are held because of their ability to facilitate routine operations of the
company. These assets are not held for purposes of achieving investment returns. 19. Answer (D) is correct. A company will hold cash and marketable securities to facilitate
Answer (A) is incorrect because most companies are not in business to earn high returns on business transactions because cash is a primary medium of exchange. Cash and near-cash
liquid assets (i.e., they are held to facilitate operations). Answer (B) is incorrect because the items are also held to meet future needs, to satisfy compensating balance requirements
holding of cash and cash-like assets is not a major factor in controlling taxes. Answer (C) is imposed by lenders, and to provide a precautionary balance for security purposes. Cash is
incorrect because investments in Treasury bonds do not have sufficient liquidity to serve as usually not held in an attempt to earn maximum returns on investment because cash and
short-term assets. Answer (D) is incorrect because common stocks do not have sufficient marketable securities are not usually the highest-paying investments.
liquidity to serve as short-term cash resources. Answer (A) is incorrect because cash is held to satisfy compensating balance requirements.
Answer (B) is incorrect because cash is held to facilitate transactions. Answer (C) is incorrect
16. A very efficient cash management system could allow a firm to operate with positive net float because cash is held to meet future needs.
where the firm has a negative checkbook balance at most times but still does not bounce its
checks. The other statements are false. A good cash management system maximizes 20. Net float = Disbursements float - Collections float; therefore the larger the disbursements float
disbursement float and minimizes collections float. A well-designed lockbox system minimizes and the lower the collections float the better the cash management system. A lockbox is used
collections float which would increase a firm’s net float. Increases in interest rates raise the to speed cash collections. If a firm’s outflows come due early in the month rather than
opportunity cost of idle cash. A firm prefers to write checks, maximizing its disbursement float uniformly this will necessitate a large line of credit.
and increasing its net float.
21. Answer (D) is correct. A lock-box system accelerates the inflow of funds. A company maintains
17. Answer (D) is correct. John Maynard Keynes, founder of Keynesian economics, concluded mail boxes, often in numerous locations around the country, to which customers send
that there were three major motives for holding cash: for transactional purposes as a medium payments. A bank checks these mailboxes several times a day, and funds received are
of exchange, precautionary purposes, and speculative purposes (but only during deflationary immediately deposited to the company's account without first being processed by the
periods). company's accounting system, thereby hastening availability of the funds.
Answer (A) is incorrect because the three major motives for holding cash, according to Answer (A) is incorrect because a lock-box system is not related to compensating balances; a
Keynesian economics, are for transactional, precautionary, and speculative purposes. Answer compensating balance may be required by a covenant in a loan agreement that requires a
(B) is incorrect because the three major motives for holding cash, according to Keynesian company to maintain a specified balance during the term of the loan. Answer (B) is incorrect
economics, are for transactional, precautionary, and speculative purposes. Answer (C) is because a lock-box system is a process by which payments are sent to a bank's mailbox,
RPCPA, AICPA, CMA & CIA EXAMINATION QUESTIONS Page 24 of 36
MANAGEMENT ADVISORY SERVICES WORKING CAPITAL FINANCE

which is checked during normal post office hours. Answer (C) is incorrect because the use of Answer (B) is incorrect because a check is not involved in an EFT. Answer (C) is incorrect
a lock-box system entails sending checks through the mail to a post office box. Thus, it does because an ACH transfer involves the actual transfer of funds electronically; it is not just a
not reduce the risk of losing checks in the mail. computer generated document. Answer (D) is incorrect because an EFT is not a check-like
instrument.
22. REQUIRED: The working capital technique that delays the outflow of cash.
DISCUSSION: A draft is a three-party instrument in which one person (the drawer) orders a 25. Answer (C) is correct. Various methods of accelerating cash collections include decentralized
second person (the drawee) to pay money to a third person (the payee). A check is the most collection outposts (normally one in each Federal Reserve District), electronic funds transfers,
common form of draft. It is an instrument payable on demand in which the drawee is a bank. centralized banking for all company branches to avoid having to maintain minimum balances
Consequently, a draft can be used to delay the outflow of cash. A draft can be dated on the in several locations, and lockbox systems. A compensating balance is a minimum average or
due date of an invoice and will not be processed by the drawee until that date, thereby absolute amount that must be maintained in a bank account. Hence, it is not a means of
eliminating the necessity of writing a check earlier than the due date or using an EFT. Thus, accelerating cash collections. This requirement means that less cash is available to the
the outflow is delayed until the check clears the drawee bank. depositor.
Answer (A) is incorrect because factoring is the sale of receivables and therefore concerns Answer (A) is incorrect because it is a common method of accelerating cash collections.
cash inflows, not outflows. Answer (B) is incorrect because a lockbox system is a means of Answer (B) is incorrect because it is a common method of accelerating cash collections.
accelerating cash inflows. Answer (D) is incorrect because an electronic funds transfer results Answer (D) is incorrect because it is a common method of accelerating cash collections.
in an immediate deduction from the payor’s bank account, thereby eliminating float.
26. Answer (D) is correct. Compensating balances are either (1) an absolute minimum balance or
23. Answer (B) is correct. Payment by draft, a three-party instrument in which the drawer orders (2) a minimum average balance that bank customers must keep at the bank. These are
the drawee to pay money to the payee, is a means of slowing cash outflows. A check is the generally required by the bank to compensate for the cost of services rendered. Maintaining
most common type of draft. Check float arises from the delay between an expenditure and the compensating balances will not accelerate a company's cash inflows because less cash will be
clearing of the check through the banking system. available even though the amount of cash coming in remains unchanged.
Answer (A) is incorrect because concentration banking, a lockbox system, and the use of a Answer (A) is incorrect because multiple collection centers throughout the country will reduce
local post office box are techniques used to accelerate cash receipts. Answer (C) is incorrect the time required to receive cash in the mail. For example, California customers of a New York
because EDI is the communication of electronic documents directly from a computer in one firm would make payment to a West Coast center. Thus, the company would receive the cash
entity to a computer in another entity. Thus, EDI expedites cash payments. The payee two or three days sooner. Answer (B) is incorrect because direct deposit by customers into a
receives the money almost instantaneously. Answer (D) is incorrect because concentration lock-box also speeds cash into company accounts. Answer (C) is incorrect because special
banking, a lockbox system, and the use of a local post office box are techniques used to handling of large checks is a cost-effective way to deposit large amounts.
accelerate cash receipts.
27. Answer (A) is correct. Cash conversion is the process of converting cash to products and back
24. REQUIRED: The definition of an automated clearing house (ACH) electronic transfer. to cash. The length of the cycle is from the cash outflow to the cash inflow. Thus, the cash
DISCUSSION: (A) An ACH electronic funds transfer (EFT) is an electronic payment to a conversion cycle, in the case of production, begins with the cash purchase of raw materials
company’s account of a concentration bank. A concentration bank is a large bank to which a and ends with the collection of accounts receivable.
company transfers funds from local depository banks. These local banks operate the Answer (B) is incorrect because the cash inflows have not been received. Answer (C) is
company’s lockboxes and thus serve as collection points. The transfer of funds to the incorrect because the cash inflows have not been received. Answer (D) is incorrect because
concentration bank allows the company to take advantage of economies of scale in cash the cash inflows have not been received.
management. The use of an ACH facilitates concentration banking. ACHs are electronic
networks operated by the Federal Reserve (except for the New York regional ACH 28. Statement a is false. If inventory increases, and sales do not, more cash is being “tied up” in
association) that guarantee 1-day clearing. inventory so the cash conversion cycle is increased, not reduced. Statement b is true. If the
RPCPA, AICPA, CMA & CIA EXAMINATION QUESTIONS Page 25 of 36
MANAGEMENT ADVISORY SERVICES WORKING CAPITAL FINANCE

company reduces its DSO, it is collecting its accounts receivables more efficiently, so it
reduces the cash conversion cycle. Statement c is false. If the company pays its bills sooner, it 33. Answer (A) is correct. The optimal amount of cash to be raised by selling securities is
uses its cash to pay off accounts payable, which increase its cash conversion cycle. calculated by a formula similar to that used to determine the economic order quantity for
inventory.
29. Statements a and b are correct; therefore, statement d is the appropriate choice. Delaying 2(F)(T)
payments to suppliers increases the length of the cash conversion cycle. C+ =
k
30. Answer (D) is correct. Cash and short-term investments are crucial to a firm's continuing If: C+ = Cash to be raised
success. Sufficient liquidity must be available to meet payments as they come due. At the T = Total cash needed for the period
same time, liquid assets are subject to significant control risk. Therefore, liquidity and safety F = Cost of making a securities trade
are the primary concerns of the treasurer when dealing with highly liquid assets. Cash and k = Opportunity cost of holding cash
short-term investments are held because of their ability to facilitate routine operations of the The optimal amount of cash to be raised by selling securities is inversely related to the rate of
company. These assets are not held for purposes of achieving investment returns. return forgone (opportunity cost) and directly related to the cost of the transaction.
Answer (A) is incorrect because most companies are not in business to earn high returns on Answer (B) is incorrect because a high (low) opportunity cost results in a lower (higher)
liquid assets (i.e., they are held to facilitate operations). Answer (B) is incorrect because the optimal cash balance, whereas high (low) transaction costs result in a higher (lower) optimal
holding of cash and cash-like assets is not a major factor in controlling taxes. Answer (C) is cash balance. Answer (C) is incorrect because a high (low) opportunity cost results in a lower
incorrect because investments in Treasury bonds do not have sufficient liquidity to serve as (higher) optimal cash balance, whereas high (low) transaction costs result in a higher (lower)
short-term assets. optimal cash balance. Answer (D) is incorrect because a high (low) opportunity cost results in
a lower (higher) optimal cash balance, whereas high (low) transaction costs result in a higher
31. Answer (C) is correct. The trade-off between risk and return must be considered because (lower) optimal cash balance.
liquid assets are usually less profitable than less-liquid alternatives. However, a greater
liquidity means less risk of being unable to meet obligations when they are due. 34. REQUIRED: The assumption not made in the EOQ model.
Answer (A) is incorrect because financial leverage concerns the extent to which debt financing DISCUSSION: (D) The EOQ formula is a deterministic model that requires a known demand
is used. Answer (B) is incorrect because operating leverage concerns the proportion of fixed for inventory, or in this case, the amount of cash needed. Thus, the cash flow requirements
operating costs. Answer (D) is incorrect because the costs of issuing securities relate to cannot be random. The model also assumes a given carrying (interest) cost and a flat
capital structure finance. transaction cost for converting marketable securities to cash, regardless of the amount
withdrawn.
32. REQUIRED: The true statement about cash management models. Answer (A) is incorrect because the EOQ model assumes that the cost of a transaction is
DISCUSSION: (D) Because cash and inventory are both non-earning assets, in principle they independent of the dollar amount of the transaction and interest rates are constant over the
may be treated similarly. The alternative to holding cash, however, is to hold marketable short-run. Answer (B) is incorrect because the EOQ model assumes an opportunity cost is
securities that do earn interest or dividends. Thus, a cash management model determine how associated with holding cash, beginning with the first dollar. Answer (C) is incorrect because
much of a firm’s liquidity should be held as cash and how much in the form of marketable the EOQ model assumes that the total demand for cash is known with certainty.
securities.
Answer (A) is incorrect because credit and collection policies concern receivables and are not 35. Answer (A) is correct. Risk and yield are fundamental concerns, and taxation is a pervasive
influenced by an EOQ model for inventory management. Answer (B) is incorrect because the issue that affects yield. Default risk is the risk that a borrower will be unable to make interest
level of marketable securities is in part determined by cash needs, so (D) is a better answer. payments or principal repayments on debt (e.g., bonds purchased from a financially troubled
Answer (C) is incorrect because the relationship between current assets and current liabilities company have high default risk). Taxability refers to the tax treatment of any income or loss
concerns many factors other than cash management.
RPCPA, AICPA, CMA & CIA EXAMINATION QUESTIONS Page 26 of 36
MANAGEMENT ADVISORY SERVICES WORKING CAPITAL FINANCE

from a security. Relative yields are the rates of return of each security in comparison with other mutual funds, Eurodollar market time deposits, and consumer credit loans. Capital markets
potential investments. trade shares and long-term debt.
Answer (B) is incorrect because these matters are a financial manager's overall responsibility, Answer (A) is incorrect because mortgages are long-term capital market securities. Answer (B)
not securities selection criteria. Answer (C) is incorrect because these cycles are important in is incorrect because preferred shares are long-term capital market securities. Answer (D) is
working capital management. The operating and payment cycles are components of the cash incorrect because U.S. Treasury bonds are long-term capital market securities.
conversion cycle. This cycle lasts from the time cash is paid for resources to the time cash is
received in payment for goods sold. Answer (D) is incorrect because these are factors in the 40. Answer (D) is correct. A certificate of deposit (CD) is a form of savings deposit that cannot be
determination of cash flow. The inventory conversion cycle is the time between acquisition of withdrawn before maturity without incurring a high penalty. A negotiable CD can be traded.
resources and sale of finished goods. The receivables conversion cycle lasts from the date of CDs usually have a fairly high rate of return compared with other savings instruments because
sale of finished goods to the date of cash collection. A payable deferral period is the lapse of they are for fixed, usually long-term periods. However, their yield is less than that of
time between purchase of resources and the date they are paid for. Thus, the cash conversion commercial paper and bankers' acceptances because they are less risky.
cycle does not begin until the end of the payable deferral period. Answer (A) is incorrect because negotiable CDs do have a secondary market (i.e., they are
negotiable). Answer (B) is incorrect because negotiable CDs are regulated. Answer (C) is
36. Answer (A) is correct. A Treasury bill is a short-term U.S. government obligation that is sold at incorrect because negotiable CDs are typically issued in a denomination of $100,000.
a discount from its face value. A Treasury bill is highly liquid and nearly risk-free, and it is often
held as a substitute for cash. 41. Answer (C) is correct. Of the choices given, the commercial paper of a top-rated (most
Answer (B) is incorrect because common stock lacks the liquidity necessary to be a cash creditworthy) company has the least risk. Commercial paper is preferable to stock or stock
substitute. Common stock can also be a risky investment. Answer (C) is incorrect because options because the latter represent only a residual equity in a corporation. Commercial paper
gold lacks the liquidity necessary to be a cash substitute. Gold can also be a risky investment. is debt and thus has priority over stockholders' claims. Also, commercial paper is a very short-
Answer (D) is incorrect because Aaa corporate bonds lack the liquidity necessary to be a cash term investment. The maximum maturity allowed without SEC registration is 270 days.
substitute. Aaa corporate bonds can also be risky investments. However, it can be sold only to sophisticated investors without registration.
Answer (A) is incorrect because municipal bonds are rarely considered marketable securities
37. Answer (B) is correct. The marketable securities with the lowest default risk are those issued in that they constitute long-term debt. Answer (B) is incorrect because common stock does not
by the federal government because they are backed by the full faith and credit of the U.S. have as high a priority in company assets as commercial paper or other debt. Answer (D) is
Agency securities are issued by agencies and corporations created by the federal government, incorrect because common stock does not have as high a priority in company assets as
such as the Federal Housing Administration. They are backed by a secondary promise from commercial paper or other debt.
the government.
Answer (A) is incorrect because securities issued by a federal agency are first backed by that 42. Answer (C) is correct. A small firm with surplus cash should invest for the highest return and
agency and secondarily by the U.S. government. Answer (C) is incorrect because repurchase lowest risk. The ability to convert the investment into cash without a loss of principal is also
agreements could become worthless if the organization agreeing to make the repurchase goes important. Money market mutual funds invest in money market certificates such as treasury
bankrupt. Answer (D) is incorrect because commercial paper is unsecured. bills, negotiable CDs, and commercial paper. Because of diversification, these mutual funds
are superior to any single instrument.
38
. Only Treasury issues are insured by the U. S. government; the shorter-term the instrument, the Answer (A) is incorrect because a small firm may not have enough surplus cash to invest in
safer the instrument. commercial paper, which usually consists of secured or unsecured promissory notes of large
corporations. Answer (B) is incorrect because the transactions cost of bankers' acceptances
39. Answer (C) is correct. Funds are borrowed or lent for short periods (less than one year) in is high. A banker's acceptance is a unique credit instrument used to finance both domestic and
money markets. Examples of instruments traded in money markets are U.S. Treasury bills, international "self-liquidating" transactions. It is usually initiated by a bank's irrevocable letter of
bankers' acceptances, commercial paper, negotiable certificates of deposit, money market credit on behalf of the bank's customer, on which the company doing business with the bank's
RPCPA, AICPA, CMA & CIA EXAMINATION QUESTIONS Page 27 of 36
MANAGEMENT ADVISORY SERVICES WORKING CAPITAL FINANCE

customer draws a time draft. The company discounts the time draft with the company's local incorrect because Eurodollar deposits tend to be for larger, not smaller, amounts.
bank and receives immediate payment. The local bank forwards the time draft to the bank Furthermore, smaller deposits tend to earn lower, not higher, rates than larger deposits.
customer for payment. Answer (D) is incorrect because an increase in interest rates could
cause a substantial loss in principal. 46. Answer (D) is correct. Marketable securities are near-cash items used primarily for short-term
investment. Examples include U.S. Treasury bills, Eurodollars, commercial paper, money-
43. Answer (B) is correct. Eurobonds are, by definition, always sold in some country other than the market mutual funds with portfolios of short-term securities, bankers' acceptances, floating-
one in whose currency the bond issue is denominated. Their advantage is that they are rate preferred stock, and negotiable CDs of U.S. banks. A convertible bond is not a short-term
customarily less stringently regulated than most other bonds. Hence, transaction costs are investment because its maturity date is usually more than one year in the future and its price
lower. can be influenced substantially by changes in interest rates or by changes in the investee's
Answer (C) is incorrect because foreign bonds are denominated in the currency of the country stock price.
in which they are sold. (A) is incorrect because Eurobonds are not always denominated in Answer (A) is incorrect because U.S. Treasury bills are short-term marketable securities.
Eurodollars, which are U.S. dollars deposited outside the U.S. Answer (D) is incorrect Answer (B) is incorrect because Eurodollars are short-term marketable securities. Answer (C)
because Eurobonds are usually issued not as registered bonds but as bearer bonds, so is incorrect because commercial paper is a short-term marketable security.
names and nationalities of the investors are not recorded.
47. REQUIRED: The definition of “short-selling”
44. Answer (C) is correct. Eurodollars are U.S. dollars on deposit in a foreign bank. These DISCUSSION: (B) Short-selling is accomplished by borrowing securities from a broker and
deposits are created when a check is drawn on a dollar deposit in a U.S. bank and then selling those securities. At a later time, the loan is repaid by buying securities on the open
deposited in a bank outside the U.S. This amount is then available for lending by the foreign market and returning them to th broker. The seller speculates that the stock’s market price will
bank to its customers. However, the depositors still hold claims denominated in dollars. decline.
Because Eurodollars are outside the direct control of the U.S. monetary authorities, U.S. Answer (A) is incorrect because margin trading involves buying securities bu borrowing money
banking regulations with respect to reserves, insurance, interest ceilings, etc., do not apply. from a broker. Answer (C) is incorrect because the investor does not own the shares sold in a
The absence of these costs means that Eurodollar deposit rates tend to be higher, not lower, short sale. Answer (D) is incorrect because the short seller is betting that the stock will
than domestic U.S. rates on equivalent instruments. decrease in price.
Answer (A) is incorrect because these deposits are outside the control of U.S. authorities
because they are in banks outside the U.S. Answer (B) is incorrect because these deposits 48. Answer (C) is correct. The day's sales outstanding (days of receivables) may be stated as the
are outside the control of U.S. authorities because they are in banks outside the U.S. Answer accounts receivable balance divided by average credit sales per day or as days in the year
(D) is incorrect because interest rates on these deposits, and loans made thereon, are tied to divided by the receivables turnover. It is the average time required to convert the enterprise's
the LIBOR, which is the rate paid on deposits of other large banks by the largest London receivables into cash. Thus, it is also called the receivables collection period.
banks with the highest credit standing. Answer (A) is incorrect because the inventory conversion period (days of inventory) is the
average time required to convert materials into finished goods and then to sell them. This
45. Answer (D) is correct. Eurodollars are U.S. dollars deposited in banks outside the U.S. process typically occurs before the receivables collection period, and the amount of time in
Because it is outside the direct control of the U.S. monetary authorities, the Eurodollar market one period does not necessarily bear any relationship to the other. Answer (B) is incorrect
has lower costs. For example, U.S. reserve requirements and FDIC premium payments do not because the cash conversion cycle equals the inventory conversion period, plus the
apply in this market. A lower cost market can offer depositors higher interest rates. receivables collection period, minus the payables deferral period (average time between
Answer (A) is incorrect because Eurodollar borrowers tend to pay lower, not higher, rates. resource purchases and payment of cash for them). It estimates the time between when the
Borrowers and depositors can both receive more favorable rates because, with its lower costs, enterprise makes payments and when it receives cash inflows. Answer (D) is incorrect
the Eurodollar market can offer smaller spreads between borrowing and lending rates. because the inventory divided by the sales per day is the inventory conversion period (days of
Answer (B) is incorrect because U.S. dollars are on deposit in both cases. Answer (C) is inventory).
RPCPA, AICPA, CMA & CIA EXAMINATION QUESTIONS Page 28 of 36
MANAGEMENT ADVISORY SERVICES WORKING CAPITAL FINANCE

Answer (C) is incorrect because, most likely, higher quality accounts will mean a shorter
49. REQUIRED: The meaning of a firm’s average collection period. average collection period.
DISCUSSION: (A) The average collection period measures the number of days between the
date of sale and the date of collection. It should be related to a firm’s credit terms. For 53. REQUIRED: The calculation of the average gross receivable balance.
example, a firm that allows terms of 2/15, net 30 should have an average collection period of DISCUSSION: A firm’s average gross receivables balance can be calculated by multiplying
somewhere between 15 and 30 days. average daily sales by the average collection period (days’ sales outstanding). Alternatively,
Answer (B) is incorrect because it describes the concept of float. Answer (C) is incorrect annual credit sales can be divided by the accounts receivable turnover (net credit sales ÷
because the average collection period includes the total time before a payment is received. average accounts receivable) to obtain the average balance in receivables.
Including the periods both before and after the end of the normal credit period. Answer (D) is Answers (A) and (B) are incorrect because Alternative 1 cannot be correct. Neither of the
incorrect because it describes the normal credit period. multiplicands is a dollar figure, sot the product could not be the dollar balance of receivables.
Answer (D) is incorrect because Alternative III cannot be correct. It contains average gross
50. REQUIRED: The item measured by an aging of accounts receivable. receivables, the amount being calculated.
DISCUSSION: (D) The purpose of an aging of receivables is to classify receivables by due
date. Those that are current (not past due) are listed in one column, those less than 30 days 54. Answer (C) is correct. A high receivables turnover is indicative of short credit terms and
past due in another column, etc. The amount in each category can then be multiplied by an possibly the use of a lockbox system to speed up cash flows.
estimated bad debt percentage that is based on a company’s credit experience and other Answer (A) is incorrect because a lenient credit policy would lead to a low turnover. Answer
factors. The theory is that the oldest receivables are the least likely to be collectible. Aging (B) is incorrect because small discounts although helpful, would be unlikely to lead to an
the receivables and estimating the uncollectible amounts is one method of arriving at the unusually high turnover. Small discounts would not be as helpful as a lockbox system and
appropriate balance sheet valuation of the accounts receivable account. short credit terms. Answer (D) is incorrect because a high turnover would be indicative of an
Answer (A) is incorrect because an aging schedule is used for receivables, not liabilities. efficient credit and collection department.
Answer (B) is incorrect because an aging schedule concerns specific accounts, not averages.
Answer (C) is incorrect because an aging schedule focuses on uncollectible receivables. 55. Answer (B) is correct. A decline in the receivable turnover indicates that either sales are
declining relative to receivables, or receivables are increasing relative to sales. Thus, the firm
51. Answer (B) is correct. A quantity discount is an attempt to increase sales by reducing the unit is collecting credit sales more slowly than before.
price on bulk purchases. It concerns only the price term of an agreement, not the credit term, Answer (A) is incorrect because sales are being collected more slowly when the turnover
and thus is unrelated to credit and collection policy. declines. Answer (C) is incorrect because an increase in sales will increase the turnover ratio.
Answer (A) is incorrect because the quality of accounts is important to credit policy since it is Answer (D) is incorrect because inventories have no impact on the receivable turnover ratio.
inversely related to both sales and bad debts. Answer (C) is incorrect because offering a cash
discount improves cash flow and reduces receivables and the cost of extending credit. 56. Answer (D) is correct. The accounts receivable turnover ratio equals net credit sales divided by
Answer (D) is incorrect because the level of collection expenditures must be considered when average receivables. Hence, it will decrease if a company lengthens the credit period or the
implementing a collection policy. The marginal cost of a credit and collection policy should not discount period because the denominator will increase as receivables are held for longer
exceed its revenue. times.
Answer (A) is incorrect because write-offs do not reduce net receivables (gross receivables -
52. Answer (D) is correct. Increasing the quality of the accounts rejected means that fewer sales the allowance) and will not affect the receivables balance and therefore the turnover ratio if an
will be made. The company is therefore not trying to maximize its sales or increase its bad allowance system is used. Answer (B) is incorrect because a decline in sales near the end of
debt losses. The objective is to reduce bad debt losses and thereby maximize profits. the period signifies fewer credit sales and receivables, and the effect of reducing the
Answer (A) is incorrect because tightening credit will reduce sales and bad debt losses. numerator and denominator by equal amounts is to increase the ratio if the fraction is greater
Answer (B) is incorrect because tightening credit will reduce sales and bad debt losses.
RPCPA, AICPA, CMA & CIA EXAMINATION QUESTIONS Page 29 of 36
MANAGEMENT ADVISORY SERVICES WORKING CAPITAL FINANCE

than 1.0. Answer (C) is incorrect because an increase in cash sales with no diminution of DISCUSSION: (A) An increase in discounts taken accompanied by declines in receivables
credit sales will not affect receivables. balances and doubtful accounts all indicate that collections on the increased sales have been
accelerated. Accordingly, the average collection period is a ratio calculated by dividing the
57. REQUIRED: The reason for offering credit terms of 2.10, net 30. number of days in a year (365) by the receivable turnover. Thus, the higher the turnover, the
DISCUSSION: (D) Because these terms involve an annual interest cost of over 36%, a shorter the average collection period. The turnover increases when either sales (the
company would not offer them unless it desperately needed cash. Also, credit terms are numerator) increase or receivables (the denominator) decrease. Accomplishing both higher
typically somewhat standardized within an industry. Thus if most companies in the industry sales and a lower receivables increases the turnover and results in a shorter collection period.
offer similar terms, a firm will likely be forced to match the competition or lose market share. Answer (B) is incorrect because a decrease in the percentage discount offered provides no
Answer (A) is incorrect because, if the company does not need cash, it would not offer cash incentive for early payment. Answer (C) is incorrect because accounts receivable turnover
discounts, regardless of its cost of capital, unless required to match competition. Answer (B) (sales ÷ average receivables) has increased. Answer (D) is incorrect because no information
is incorrect because the ability to borrow at a lower rate is a reason for not offering cash is given relative to working capital elements other than receivables. Both receivables and
discounts. Answer (C) is incorrect because the relationship between the cost of capital and cash are elements of working capital, so an acceleration of customer payments will have no
the prime rate may not be relevant if the firm cannot borrow at the prime rate. effect on working capital.

58. REQUIRED: The true statement about extending credit for a period longer than the 61. REQUIRED: The true statement about a change in credit policy that has resulted in greater
purchase’s operating cycle. sales and a reduction in accounts receivable.
DISCUSSION: (B) The normal operating cycle is defined as the period from the acquisition of DISCUSSION: (B) An increase in discounts taken accompanied by declines in receivables
inventory to the collection of the account receivable. If trade credit is for a period longer than balances and doubtful accounts all indicate that collections on the increased sales have been
the normal operating cycle, the seller must therefore be financing more than just the purchase accelerated. Accordingly, the average collection period must have declined. The average
of inventory. collection period is a ratio calculated by dividing the number of days in a year (365) by the
Answer (A) is incorrect because a seller that extends long-term credit will have a higher level receivable turnover. Thus, the higher the turnover, the shorter the average collection period.
of receivables than a firm with a shorter credit period. Answer (C) is incorrect because the The turnover increases when either sales (the numerator) increase, or receivables (the
seller is not guaranteed that a purchaser will resell the merchandise. Answer (D) is incorrect denominator) decrease. Accomplishing both higher sales and a lower receivables increases
because offering a discount may accelerate payment. the turnover and results in a shorter collection period.
Answers (A) and (C) are incorrect because no statement can be made with respect to profits
59. Answer (C) is correct. If the cause of increased sales is an increase in the cash discount, it without knowing costs. Answer (D) is incorrect because the discount may have been
can be inferred that the additional customers would pay during the discount period. Thus, cash increased, which has led to quicker payments. Answer (E) is incorrect because the bad debt
would be collected more quickly than previously and the cash conversion cycle would be loss percentage has apparently declined. Sales are up; doubtful accounts are fewer.
shortened.
Answer (A) is incorrect because the operating cycle would decrease since the average time 62. REQUIRED: The factor not considered in an analysis of propose credit policies.
from cash disbursement to cash realization would be shorter. Answer (B) is incorrect because DISCUSSION: (B) All factors should be considered that differ between the two policies.
the average collection period would decrease since the average time from cash disbursement Factors that do not differ, such as the current bad debt experience, are not relevant. Ryan
to cash realization would be shorter. Answer (D) is incorrect because more customers will must estimate the expected bad debt losses under each new policy.
take discounts. Answer (A) is incorrect because the cost of funds is an obvious element in the analysis of any
investment. Answer (C) is incorrect because the impact on the current customer base of
60. REQUIRED: The true statement about a change in credit policy that has resulted in greater extending terms to only certain customers is relevant. The current customers may demand
sales and a reduction in accounts receivable. the same terms. Answer (D) is incorrect because existing loan agreements may require Ryan

RPCPA, AICPA, CMA & CIA EXAMINATION QUESTIONS Page 30 of 36


MANAGEMENT ADVISORY SERVICES WORKING CAPITAL FINANCE

to maintain certain ratios at stated levels. Thus, Ryan’s ability to increase receivables and refinancing. Because the debtor company will be forced to meet principal and interest
possible bad debt losses may be limited. payments quickly, perhaps before expected funds from a new project, the danger of default is
increased. Also, future interest rates are difficult to predict.
63. Answer (A) is correct. Relaxing the credit policy for customers will lead to increased sales Answer (B) is incorrect because leverage is the use of borrowed funds to earn returns for
because more people will be eligible for more credit. As sales increase, purchase orders will stockholders. It is irrelevant whether the borrowed funds are long- or short-term. Answer (C) is
increase to accommodate the higher sales levels. incorrect because the length of a loan does not affect the amount of liquid assets. Both long-
Answer (B) is incorrect because inventory should be increased to accommodate higher sales and short-term loans result in liquid assets. Answer (D) is incorrect because an increase in
levels. Answer (C) is incorrect because safety stock is based on expected sales, which are current liabilities decreases the current ratio.
expected to rise. Answer (D) is incorrect because a just-in-time system is not used when a
company orders inventory once a quarter. 67. Under normal conditions the yield curve is upward sloping, thus, short-term interest
rates are lower than long-term interest rates. Consequently, a firm financing with short-
64. Answer (A) is correct. A factor purchases a company's accounts receivable and assumes the term debt will pay less interest than a firm financing with long-term debt--increasing its
risk of collection. The seller receives money immediately to reinvest in new inventories. The ROE. However, a firm increases its risk by financing with short-term debt because such
financing cost is usually high: about 2 points or more above prime, plus a fee for collection. debt must be “rolled over” frequently, and the firm is exposed to the volatility of short-
Factoring has been traditional in the textile industry for years, and recently companies in many term rates. The other statements are false.
industries have found it an efficient means of operation. A company that uses a factor can
eliminate its credit department, accounts receivable staff, and bad debts. These reductions in 68. REQUIRED: The item that provides a spontaneous source of financing.
costs can more than offset the fee charged by the factor, which can often operate more DISCUSSION: (A) Trade credit is a spontaneous source of financing because it arises
efficiently than its clients because of the specialized nature of its service. automatically as part of a purchase transaction. Because of its ease in use, trade credit is the
Answer (B) is incorrect because factoring is a source of short-term funds through sale of largest source of short-term financing for may firms both large and small.
receivables. Answer (C) is incorrect because factoring is a source of short-term funds through Answers (B) and (D) are incorrect because mortgage bonds and debentures do not arise
sale of receivables. Answer (D) is incorrect because factoring is a source of short-term funds automatically as a result of a purchase transaction. Answer (C) is incorrect because the use
through sale of receivables. of receivables as a financing source requires an extensive factoring arrangement and often
involves the creditor’s evaluation of the credit ratings of the borrower’s customers.
65. Answer (B) is correct. Pledging accounts receivable is an arrangement in which receivables
are used as security for a loan. But if the receivables are not paid, the secured party still may 69. Answer (D) is correct. Trade credit is a spontaneous source of financing because it arises
proceed against the borrowers. Lenders in these circumstances choose the collateral with automatically as part of the purchase transaction.
care, retain the right of recourse against the borrower in case of default, and usually lend Answer (A) is incorrect because it occurs as a result of transactions apart from purchase
some amount less than the face value of the receivables. transactions. In other words, such credit is arranged separately from the transactions to
Answer (A) is incorrect because factoring is the outright sale of receivables for cash at a acquire the assets being financed. Answer (B) is incorrect because it occurs as a result of
discount. Answer (C) is incorrect because receivables may be sold with or without recourse. transactions apart from purchase transactions. In other words, such credit is arranged
Answer (D) is incorrect because hedging is the process of protecting oneself against loss separately from the transactions to acquire the assets being financed. Answer (C) is incorrect
because of future price changes. For example, a party that has agreed to deliver a commodity because prepaid interest is not a source of financing.
at a certain date may enter into a hedging contract to buy an equal quantity on the same date.
Any loss on one transaction should then be offset by the gain on the other. 70. Answer (C) is correct. Trade credit is a spontaneous source of financing because it arises
automatically as part of a purchase transaction. Because of its ease in use, trade credit is the
66. Answer (A) is correct. An increase in the proportion of short-term financing will not affect a largest source of short-term financing for many firms both large and small.
company's degree of leverage, but risk is increased because of the need for frequent
RPCPA, AICPA, CMA & CIA EXAMINATION QUESTIONS Page 31 of 36
MANAGEMENT ADVISORY SERVICES WORKING CAPITAL FINANCE

Answer (A) is incorrect because installment loans are usually a longer-term source of financing
and are more difficult to acquire than trade credit. Answer (B) is incorrect because commercial 74. REQUIRED: The true statement about cash discounts.
paper is normally used only by large companies with high credit ratings. Answer (D) is DISCUSSION: (D) Payments should be made within the discount periods if the cost of not
incorrect because mortgage bonds are a long-term source of financing. taking discounts exceed the firm’s cost of capital. For example, failing to take a discount when
terms are 2/10, net 30 means that the firm is paying an effective annual interest rate
71. REQUIRED: The true statement about trade credit. exceeding 36%. Thus the cost of not taking the discount is usually higher than the cost of a
DISCUSSION: (C) Trade credit is a spontaneous source of financing because it arises bank loan.
automatically as part of a purchase transaction. The terms of payment are set by the supplier, Answer (A) is incorrect because the cost of not taking a discount when terms are 2/10, net 30
but trade credit usually requires payment within a short period of time. Trade credit is an exceeds 36% annually, which is higher than the prime rate has ever been. Answer (B) is
important source of credit for all businesses but especially for buyers, such as small incorrect because the buyer is paying the amount of discount not taken in exchange for the
businesses but especially for buyers, such as small businesses, that might not have access to extra 45 days of credit. Answer (C) is incorrect because paying 2% for 20 days of credit is
other credit markets. Like all forms of financing, trade credit is subject to the risk of buyer more expensive than paying 2% for 50 days of the same amount of credit.
default.
Answer (A) is incorrect because trade credit is an important source of financing for small firms. 75. Answer (C) is correct. A small retail store would not have access to major capital markets. In
Answer (B) is incorrect because trade credit is ordinarily short-term source of financing. fact, the only options available, outside of owner financing, are bank loans and a line of credit
Answer (D) is incorrect because the cost of trade credit depends on the credit terms and the from suppliers. It is this latter alternative that is most often used because it permits the store to
price paid. A seller with generous payment terms may charge a higher price for its finance inventories for 30 to 60 days without incurring interest cost. A line of credit is an
merchandise. arrangement between a bank and a borrower in which the bank commits itself to lend up to a
certain maximum amount to the borrower in a given period.
72. Answer (A) is correct. The first term of the formula represents the periodic cost of the trade Answer (A) is incorrect because only large companies with excellent credit ratings have
discount, calculated as the cost per unit of trade credit (discount %) divided by the funds made access to the commercial paper market. Answer (B) is incorrect because a retail store must
available by not taking the discount (100 - discount %). The second term represents the have instant access to its inventory to provide continuous services to customers. Thus, a
number of times per year this cost is incurred. The multiple of these terms is the approximate terminal warehouse receipt loan would not be suitable because the inventory would not be in
annual percentage cost of not taking the trade discount. A precise formula would incorporate the immediate possession of the seller. Answer (D) is incorrect because a chattel mortgage is
the effects of compounding when calculating the annual cost. most often used for financing movable equipment. It is not well-suited to financing inventory of
Answer (B) is incorrect because the denominator of the first term should represent the funds a small retailer with high turnover because of the difficulty of identification.
made available by not taking the discount (100 - discount %). Answer (C) is incorrect because
the first term is the reciprocal of the correct term. Answer (D) is incorrect because the second 76. Statement b illustrates an aggressive financing policy, not a conservative one.
term is the reciprocal of the correct term.
77. Answer (D) is correct. The first step is to determine the actual annual percentage interest rate
73. Answer (D) is correct. If the discount period is longer, the days of extra credit obtained by for each of the four options. Assuming a $100 invoice, the Fort Company discount represents
forgoing the discount are fewer. Assuming other factors are constant, the result is that the cost interest of $1 on a loan of $99 for 20 days (30-day credit period - 10-day discount period). The
of trade credit, that is, the cost of not taking the discount, is greater. annual interest rate is 18.1818% [(360/20) periods x ($1/$99)]. The Riley Company discount
Answer (A) is incorrect because the lower the discount percentage, the lower the opportunity represents an interest charge of $2 on a loan of $98; i.e., by not paying on the 15th day, the
cost of forgoing the discount and using the trade credit financing. Answer (B) is incorrect company will have the use of $98 for 45 days (60-day credit period - 15-day discount period).
because percentage financing cost is unaffected by the purchase price of the items. Answer The number of periods in a year would be 8 (360/45). The interest would be 16.326% ($2/$98
(C) is incorrect because percentage financing cost is unaffected by the purchase price of the x 8 periods). The Shad loan would be for $97 at a cost of $3. The loan would be for 75 days
items. (90 - 15). Given 4.8 interest periods in a year (360/75), the annual interest rate would be
RPCPA, AICPA, CMA & CIA EXAMINATION QUESTIONS Page 32 of 36
MANAGEMENT ADVISORY SERVICES WORKING CAPITAL FINANCE

14.845% ($3/$97 x 4.8). The bank loan was quoted at 14.75% on a discount basis. On a $100 will be much higher than the prime rate. Answer (C) is incorrect because the prime rate is a
note, the borrower would only receive $85.25, giving an interest rate of 17.302% bank loan rate, not the rate on commercial paper.
($14.75/$85.25). Thus, not paying Shad, Inc.'s invoices on time would be the lowest cost
source of capital, at a cost of 14.845%. 80. Answer (C) is correct. When a firm borrows money from the bank, it is often required to keep a
Answer (A) is incorrect because the actual annual percentage rate based on forgoing Shad's certain percentage of the funds in the bank at all times. These compensating balances
discount is 14.845%. This is lower than the rate on the bank loan (17.302% because it is a effectively increase the rate of interest on the money borrowed from the bank.
discount loan), or the cost of forgoing the discounts allowed by Fort Co. (18.182%) and Riley Answer (A) is incorrect because the floating interest rate is not always higher. It should float up
Co. (16.326%). Answer (B) is incorrect because the actual annual percentage rate based on or down with the prime rate. Answer (B) is incorrect because a restriction on a new issuance
forgoing Shad's discount is 14.845%. This is lower than the rate on the bank loan (17.302% does not raise the interest rate on money previously borrowed. Answer (D) is incorrect
because it is a discount loan), or the cost of forgoing the discounts allowed by Fort Co. because, if a firm chooses not to use its full line of credit and is not charged for the unused
(18.182%) and Riley Co. (16.326%). Answer (C) is incorrect because the actual annual portion, the rate of interest on the portion used does not increase.
percentage rate based on forgoing Shad's discount is 14.845%. This is lower than the rate on
the bank loan (17.302% because it is a discount loan), or the cost of forgoing the discounts 81. Answer (B) is correct. Simple interest is charged on the amount actually paid to the borrower.
allowed by Fort Co. (18.182%) and Riley Co. (16.326%). If interest is charged on a simple basis, the full $20,000 face value of the loan is made
available to the borrower.
78. REQUIRED: The true statement about short-term interest rates. Answer (A) is incorrect because interest is charged on a discount basis when it is deducted
DISCUSSION: (A) Historically, one facet of the term structure of interest rates (the from the face value borrowed. Answer (C) is incorrect because interest charged on a discount
relationship of yield and time to maturity) is that short-term interest rates have ordinarily been basis results in a deduction from the face value. The borrower does not receive the full face
lower than long-term rates. One reason is that less risk is involved in the short run. Moreover, value of the loan. Answer (D) is incorrect because interest is charged on an add-on basis
future expectations concerning interest rates affect the term structure. Most economists when the face value of the loan initially equals the borrowed amount plus the nominal interest
believe that a long-term interest rate is an average of future expected short-term interest rates. charge.
For this reason, the yield curve will slope upward if future rates are expected to rise, downward
if interest rates are anticipated to fall, and remain flat if investors think the rate is stable. 82. Answer (B) is correct. A minimum checking account balance that a firm must maintain with a
Future inflation is incorporated into this relationship. Another consideration is liquidity commercial bank is a compensating balance. A bank may require a borrower to keep a certain
preference. Investors in an uncertain world will accept lower rates on short-term investments percentage of the face value of a loan in the firm's account. This requirement raises the real
because of their greater liquidity, whereas business debtors often prefer to pay higher rates on rate of interest to the borrower.
long-term debt to avoid the hazards of short-term maturities. Answer (A) is incorrect because the cash balance necessary for a firm to conduct day-to-day
Answer (B) is incorrect because short-term rates are usually lower than long-term rates. business is a transactions balance. Answer (C) is incorrect because a cash balance held in
Answer (C) is incorrect because short-term rates are more likely to be greater than long-term reserve for random, unforeseen fluctuations in cash inflows and outflows is a precautionary
rates if current levels of inflation are high. Answer (D) is incorrect because long-term rates balance. Answer (D) is incorrect because a cash balance that is held to enable the firm to take
may be viewed as short-term rates adjusted by a risk factor. advantage of any bargain purchases that might arise is a speculative balance.

79. Answer (D) is correct. The prime interest rate is the rate charged by commercial banks to their 83. Answer (A) is correct. Banks sometimes require a borrower to keep a certain percentage of
best (the largest and financially strongest) business customers. It is traditionally the lowest rate the face amount of a loan in a non-interest-bearing checking account. This requirement raises
charged by banks. However, in recent years, banks have been making loans at still lower rates the effective rate of interest paid by the borrower. This greater rate compensates a bank for
in response to competition from the commercial paper market. services provided and results in greater profitability for the financial institution.
Answer (A) is incorrect because the prime rate has nothing to do with a commitment fee on a Answer (B) is incorrect because, in financial accounting, a valuation allowance is used to
bank loan. Answer (B) is incorrect because the effective rate on most companies' bank loans reflect losses on marketable securities. Answer (C) is incorrect because a safety stock of
RPCPA, AICPA, CMA & CIA EXAMINATION QUESTIONS Page 33 of 36
MANAGEMENT ADVISORY SERVICES WORKING CAPITAL FINANCE

inventory is held to avoid inventory stockouts. Answer (D) is incorrect because large property such as equipment or livestock). Factoring is a form of financing in which receivables
depositors may receive favorable treatment, but compensating balances are funds maintained serve as security. Answer (C) is incorrect because a chattel mortgage is a loan secured by
by loan recipients for the benefit of the lender. personal property (movable property such as equipment or livestock). Also, a floating lien is
secured by property, such as inventory, the composition of which may be constantly changing.
84. Answer (C) is correct. The discounted interest rate is based on the amount borrowed but is
paid in advance. It is calculated using the following formula: 87. Answer (C) is correct. Commercial paper is the term for the short-term (typically less than 9
Interest months), unsecured, large denomination (often over $100,000) promissory notes issued by
large, creditworthy companies to other companies and institutional investors. In many
Borrowed amount - interest
instances, the maturity date is only a few days after issuance.
Answer (A) is incorrect because it is the formula for the simple interest rate for a 1-year loan.
Answer (A) is incorrect because an agency security is issued by a corporation or agency
Answer (B) is incorrect because it is the formula for the add-on installment interest for a 1-year
created by the U.S. government. Examples are government securities issued by the bodies
loan. Answer (D) is incorrect because the correct formula for the discounted interest rate is
that finance mortgages, such as the Federal National Mortgage Association (Fannie Mae).
interest divided by the borrowed amount less interest.
Answer (B) is incorrect because bankers' acceptances are drafts drawn on deposits at a bank.
The acceptance by the bank guarantees payment at maturity. They are normally used to
85. Answer (C) is correct. A time draft (trade acceptance) is a form of commercial draft because it
finance a specific transaction. Answer (D) is incorrect because a repurchase agreement
is drawn by a seller on the buyer; that is, it calls for the buyer to pay a specified amount. The
involves a secured loan to a government securities dealer. It allows the buyer to retain interest
draft and the shipping documents related to the goods are then sent to the buyer's bank, which
income although the seller-dealer can repurchase after a specified time.
transmits the draft to the buyer. The buyer accepts the draft by signing it. A time draft,
however, is similar to a promissory note because it is payable at a specific time in the future
88. Answer (C) is correct. Commercial paper is a form of unsecured note that is sold by only the
rather than upon acceptance by the buyer, which is characteristic of a sight draft. If a seller is
most creditworthy companies. It is issued at a discount from its face value and has a maturity
reluctant to ship goods because of concern about the buyer's ability to pay a time draft, the
period of 270 days or less. Commercial paper usually carries a low interest rate in comparison
seller's bank may, for a fee, guarantee payment. This banker's acceptance is an assumption of
to other means of financing. SMA 4M, Understanding Financial Instruments, observes that no
the obligation to pay at the due date.
general (active) secondary market exists for commercial paper, but that "most dealers or
Answer (A) is incorrect because a sight draft calls for immediate payment upon delivery of the
organizations will repurchase an issue that they have sold."
shipping documents to, and the acceptance of the draft by, the buyer. Answer (B) is incorrect
Answer (A) is incorrect because commercial paper usually has a maturity date of 270 days or
because an open account is a credit arrangement involving only the signing of an invoice by
less to avoid securities registration requirements. Answer (B) is incorrect because commercial
the buyer. Answer (D) is incorrect because the description is of a conditional sales contract
paper is often issued directly by the borrowing firm. Answer (D) is incorrect because interest
except that the seller, not the bank, retains title to the goods until the buyer has completed
rates must be higher than those of Treasury bills to entice investors. Commercial paper is
payment.
more risky than Treasury bills.
86. Answer (D) is correct. An unsecured loan is a loan made by a bank based on credit
89. Answer (A) is correct. Commercial paper is a form of unsecured note that is sold by only the
information about the borrower and the ability of the borrower to repay the obligation. The loan
most creditworthy companies. It is issued at a discount from its face value and has a maturity
is not secured by collateral, but is made on the signature of the borrower. Unsecured credit is
period of less than 270 days. Commercial paper usually carries a low interest rate in
not backed by collateral. Revolving credit, bankers' acceptances, lines of credit, and
comparison to other means of financing.
commercial paper are all unsecured means of borrowing.
Answer (B) is incorrect because only large companies with good credit ratings can find buyers
Answer (A) is incorrect because a chattel mortgage is a loan secured by personal property
for their commercial paper. Answer (C) is incorrect because commercial paper is unsecured.
(movable property such as equipment or livestock). Also, a floating lien is secured by property,
Answer (D) is incorrect because investors must pay a commission similar to that on other
such as inventory, the composition of which may be constantly changing. Answer (B) is
investment securities.
incorrect because a chattel mortgage is a loan secured by personal property (movable
RPCPA, AICPA, CMA & CIA EXAMINATION QUESTIONS Page 34 of 36
MANAGEMENT ADVISORY SERVICES WORKING CAPITAL FINANCE

with a bank, that large firms often use. Answer (D) is incorrect because a line of credit is an
90. REQUIRED: The item not an advantage of using commercial paper for short-term financing. arrangement, which may be formal or informal, between a commercial bank and its customer
DISCUSSION: (C) Commercial payer is a short-term, unsecured note payable issued in large concerning the maximum loan amount available.
denominations by major companies with excellent credit ratings. Maturities usually do not
exceed 270 days. Commercial paper is a lower cost source of funds than bank loans, and no 94. Answer (B) is correct. The corporation can obtain trade credit for 20 additional days by not
compensating balances are required. Commercial paper provides a broad and efficient paying within the discount period. Instead of paying $99,000 to satisfy its obligation within 10
distribution of debt, and costly financing arrangements are avoided. The market is not open to days, it can pay $100,000 at the end of 30 days. The corporation will thus incur $1,000 in
all companies because only major corporations with high credit ratings can participate. interest to hold the $99,000 for the 20 days. Because a 360-day year has 18 such periods, the
Answers (A), (B) and (D) are incorrect because lower rates, avoidance of compensating interest rate is approximately 18.18% [($1,000/$99,000) x 18]. However, if compounding
balance requirements, and broad debt distribution are advantages of commercial paper. effects are considered, the rate is higher. The effective rate, taking compounding into
consideration, is found using the following formula:
91. Statement a is incorrect, and therefore the right answer. Commercial paper is a type of Effective rate = [1 + (1,000/99,000)]18- 1.0 = 19.83%.
unsecured promissory note issued by large, strong firms. Statements b, c, d, and e are In comparison, the 30-day note has an effective annual rate of 21.94%, calculated as follows:
all accurate statements. [1 + (.20/12)]12 - 1.0 = 21.94%
Therefore, the corporation should use trade credit to obtain the short-term credit.
92. Answer (A) is correct. A short-term security issued by a corporation or agency created by the Answer (A) is incorrect because the effective trade credit rate is 19.83% depending on the
U.S. government, such as the Federal Housing Administration, is an agency security (agency method of calculation. Answer (C) is incorrect because the note has an effective rate,
issue). Among the largest issuers of agency securities (excluding the Treasury) are the including compounding effects, of 21.94%. The following is the calculation: Effective rate = [1
Federal Home Loan Banks, the Federal National Mortgage Association (Fannie Mae), and the + (.20/12)]12- 1.0 = 21.94%. Answer (D) is incorrect because the note has an effective rate,
other entities that provide credit to farmers and home buyers. Other issuers of home including compounding effects, of 21.94%. The following is the calculation: Effective rate = [1
mortgage-backed securities include the Government National Mortgage Association (Ginnie + (.20/12)]12- 1.0 = 21.94%.
Mae) and the Federal Home Loan Mortgage Corporation (Freddie Mac).
Answer (B) is incorrect because bankers' acceptances are issued by commercial banks to 95. Answer (D) is correct. The most costly combination of characteristics is a higher compensating
finance specific transactions. Answer (C) is incorrect because commercial paper is a short- balance and discount interest. The higher the compensating balance, the higher the portion of
term, unsecured, promissory note issued by a commercial enterprise. Answer (D) is incorrect the loan funds that must be left on deposit with the lender. Hence, the interest paid is charged
because a repurchase agreement involves what is in essence a secured loan to a dealer in on a smaller amount of funds available to be used by the borrower, and the effective cost is
government securities. higher. Also, discount interest is deducted from the loan funds in advance, resulting in a further
increase in the effective financing cost.
93. Answer (B) is correct. A document of title is usually issued by a bailee covering goods in the Answer (A) is incorrect because lower compensating balances and regular interest are less
bailee's possession or care (UCC 1-201). It represents ownership of the goods and is costly. Answer (B) is incorrect because lower compensating balances and regular interest are
ordinarily needed to obtain the goods from the bailee. The two major types of documents of less costly. Answer (C) is incorrect because lower compensating balances and regular interest
title are bills of lading (issued by carriers) and warehouse receipts. A warehouse receipt is are less costly.
issued by a person engaged in the business of storing goods for hire. Security for short-term
inventory financing can be arranged if the debtor places its inventory under the control of the 96. Answer (A) is correct. The most desirable set of terms are those that result in the lowest cost
lender or its agent (e.g., a public warehouse), and the lender holds the warehouse receipts. of borrowing. Discount interest results in a higher effective borrowing cost than simple interest
Answer (A) is incorrect because commercial paper is a type of unsecured, short-term because the bank deducts interest in advance so the borrower receives less than the face
promissory note issued by large firms to other firms, insurance companies, mutual funds, etc. value of the loan. A compensating balance results in a higher effective borrowing cost because
Answer (C) is incorrect because a revolving credit agreement is a formal line of credit, usually the compensating balance is an amount of cash that the firm is unable to use. The cheapest
RPCPA, AICPA, CMA & CIA EXAMINATION QUESTIONS Page 35 of 36
MANAGEMENT ADVISORY SERVICES WORKING CAPITAL FINANCE

terms, given that all options have the same nominal interest rate, will be simple interest with no
compensating balance.
Answer (B) is incorrect because discount interest is disadvantageous to the borrower. Answer
(C) is incorrect because a compensating balance is disadvantageous to the borrower. Answer
(D) is incorrect because discount interest and a compensating balance are disadvantageous to
the borrower.

97. Answer (A) is correct. For any given quoted nominal rate, the least frequent compounding is
associated with the lowest effective annual percentage cost. Annual compounding is less
frequent than semiannual, quarterly, or monthly. The term of the loan is not relevant to the
calculation of the effective annual percentage cost of financing.
Answer (B) is incorrect because, the more frequent the interest compounding, the more costly
the loan. Semiannual, quarterly, and monthly compounding are all more frequent than annual
compounding. Answer (C) is incorrect because, the more frequent the interest compounding,
the more costly the loan. Semiannual, quarterly, and monthly compounding are all more
frequent than annual compounding. Answer (D) is incorrect because, the more frequent the
interest compounding, the more costly the loan. Semiannual, quarterly, and monthly
compounding are all more frequent than annual compounding.

98. Answer (D) is correct. If the company chooses the line of credit, it will pay 17% interest on
$80,000 ($100,000 - $20,000 discount) and 1% on the $20,000 unused portion, a total of
$13,800. The effective interest rate would thus be 17.25% ($13,800 ÷ $80,000).
Answer (A) is incorrect because if the company forgoes the cash discount, its effective rate is
25% ($20,000 ÷ $80,000 immediate cash price). Answer (B) is incorrect because an 18%
effective rate exceeds that on the line of credit. Answer (C) is incorrect because the effective
rate would be 18.75% [(15% x $100,000) ÷ $80,000 available funds].

RPCPA, AICPA, CMA & CIA EXAMINATION QUESTIONS Page 36 of 36

You might also like